This is topic The nature of science in forum Books, Films, Food and Culture at Hatrack River Forum.


To visit this topic, use this URL:
http://www.hatrack.com/ubb/main/ultimatebb.php?ubb=get_topic;f=2;t=044678

Posted by Mathematician (Member # 9586) on :
 
(This was originally going to be posted in the "Pluto's not a planet anymore" topic, but it just seemed to diverge to far from topic).

(I don't mean to call out Orincoro at all, he/she just had the most recent version of this quote)

quote:
Originally posted by Orincoro:
...the fact that ID is completely outside the realm of science.

That's a pretty strong statement.

I'd be TOTALLY willing to agree that at the moment, proponents of ID have made no scientifically falsifiable claim. I'd also be the first to point out that, at the moment, this disqualifies ID as being science (and so I agree that, at least for now, ID has no place in a science classroom).

However, as an example, string theory still hasn't made a falsifiable claim. By similar reasoning as above, this currently disqualifies it as being science. Yet, I'm ok with this being taught in a science classroom.

What's the difference?

The difference is that at the moment, there's no hope of ID coming out with scientifically testiable claims. In direct contrast, string theory seems much more likely to begin making these claims.

But what if someday, ID DOES start making testable claims (I have NO idea what something like that would look like, but my (our) ignorance certainly isn't a reason for dismissing the possibility)? Likewise, what if string theory never ends up making a testable claim? This is certainly a possibility (note, I'm in no way vouching for the probability of this happening, just noting that as it's currently understood, this is possible). What if string theory makes a testable claim, but we won't have the engineering skills to actually test it for 1000 years? 10,000 years?

I guess I'm trying to point out that the lines between philosophy (or religion) and science can sometimes be very blurred. Now, I'm not suggesting we put ID in the classroom today, but I am suggesting we don't TOTALLY dismiss it until we have a better reason to do it.

Further, assuming ID stays outside the realm of science, that means science can tell us nothing about whether ID is true or false (by definition). Note that this does NOT mean that ID is true or false, rather, it just means that science can make the judgement. So, again, for the sake of argument, what if ID is true?

I guess, what I'm fundamnetally asking is this: Is science the search for truth? Or is it the search for the best rational/observational/experimental explanation at the time (or is there a difference between the two?)

Personally, I think that science is "the search for the best rational/observational/experimental explanataion at the time." What this means is that to me, science can never tell you "this is how things are". It can only say, "This is the best (logical/rational/ observational/experimental) explanation which has been come up with so far."

What this means is that while things like ID may have no current basis in science, that, to me, isn't an arguement for whether or not it should be taught as truth (I'm not saying whether or not it should be, because I haved no idea whether or not ID is true), though I think it SHOULDN'T be taught as science.

So, what do you think? Is science truth? Or is science the best rational approximation to truth? Or something in between? Or something more?

*EDIT* - just to be clear, I do NOT want an ID vs evolution debate, though I suppose I have NO idea where the thread will actually run

[ August 28, 2006, 06:22 PM: Message edited by: Mathematician ]
 
Posted by TomDavidson (Member # 124) on :
 
quote:
ID DOES start making testable claims (I have NO idea what something like that would look like, but my (our) ignorance certainly isn't a reason for dismissing the possibility)?
I can't think of a single testable claim ID might make that wouldn't simply be another form of natural selection.

ID as just another form of selection pressure isn't a rival to evolution.
 
Posted by FlyingCow (Member # 2150) on :
 
Well, if, for example, God was discovered to exist scientifically, then I would be okay with including checking and recording his height, weight, temperature, and blood pressure as a freshman year science lab.
 
Posted by Dan_raven (Member # 3383) on :
 
The problem is that String Theory is based on scientifically provable suppositions and is wide open to scientific debate.

ID is not.

ID is debate against a scientific theory.

ID also tends to reject scientific reasoning for more basic tenets--such as God did it.
 
Posted by Mathematician (Member # 9586) on :
 
quote:
Originally posted by Dan_raven:


ID also tends to reject scientific reasoning for more basic tenets--such as God did it.

This is exactly the assumption I'm trying to have a discussion about.

If science is THE answer in the search for truth, then "rejecting scientific reasoning for more basic tenents" is clearly wrong.

But if that's not what science is, then we can't just immediately assume that sort of treatment is wrong (if God REALLY did do it, then there's no good reasong to favor scientific reasoning over the "more basic tenents", at least in this case. That said, what's our litmus test for determining when to favor scientific reasoning over more basic tenents?)
 
Posted by Dan_raven (Member # 3383) on :
 
Science was not created to combat religion. It was created to find an alternative to religious war. You say God made the world this way. They say God made the world the other way. Both claim faith as fact. How do we determine which is correct?

We either fight it out and assume God will not let the right lose, in which case we lose free will, or we develop a methodology to test the universe and see what we can prove to be correct.
 
Posted by Mathematician (Member # 9586) on :
 
I mostly agree, especially the part about science needing to be developed (though I think there are peaceful ways to compare quality/truth of religions).

Apparently, I'm having trouble being clear in exactly what I'm asking, for that I apologize.

You seem to have an assumption that when science and religion disagree, choose science (hence, you complaint about "rejecting scientific reasoning for more basic tenets".

My question is why? If we agree in the possibility that science could be wrong and religion could be right, why is religion immediately thrown out when the two conflict? Why is the "truth" of science elevated higher than the "truth" of religion? Does science even reveal truth? Or does it just find the best model of given assumptions (which could, of course, be wrong - that's how new theories arise, most often)?

(What I'm really after is an answer to the last 2 questions I asked, as well as WHY you think that way)
 
Posted by Amka (Member # 690) on :
 
About ID:

I'm just jumping back in here and haven't been in the pluto thread, so forgive me if I get at all redundant.

A lot of theists seem to think that "God did it" is enough explanation and this really bugs me. What about "How did God do it?"

The problem with ID or not ID is that if God exists, then we live in a universe where God is in the works. We have nothing to compare it against, no control. So of course, one can't say that because there is evolution there is not God. God is not supernatural because God and nature are part of the same existance, and that is what we live in. ID would like to seperate out what God is directly responsible for and I'm not sure that it possible.

About the nature of science:

Science can really only be "the search for the best rational/observational/experimental explanataion at the time". There is a great deal of truth that cannot be arrived at directly using scientific means, and this doesn't even begin to address the existance or non-existance of deity.

Heck, what is Truth? What is goodness? What is Love? What is humor? Beauty. Art.

What is it to be who we are?

Why are we here?

Why do we experience?

These are not such superfluous questions, really. Sure, a person can go their whole life without touching on them at all, but these questions are what drive us to grow and improve both as societies and as individuals. And science can't really address them.
 
Posted by scholar (Member # 9232) on :
 
A good scientific theory will have predictive power. That is the problem with ID as a scientific explanation. God did it tells me nothing about what will happen in the future.
 
Posted by Libbie (Member # 9529) on :
 
quote:
That said, what's our litmus test for determining when to favor scientific reasoning over more basic tenents?) [/QB]
When your "more basic tenet" requires unwavering belief in something that *by its very nature* can never be tested or proven, then it is NOT a more basic tenet, but rather, a more complicated one.

The reason for selecting scientific *reasoning* over religious teachings: Science is more logical (in most cases, including the evolution/ID debate). The difference is right there in the way you posed your question. Do you take reasoning and logic, or do you take a simple, that'll-do catch-all to shut up all the intelligent questioning?

I think the real problem with the great evolution/ID debate is that evolution doesn't *need* to be taught to children. Neither does ID (nor should it be, in public schools). That level of scientific instruction can be saved for higher schooling, if it's going to cause such angst among the population. Just let people learn about it when they are capable of learning about biology en masse. It all falls into place and makes sense once you can take evolutionary theory in a complete biology course - and you can decide for yourself at that age whether you believe it's total bunk, or you believe that it's directed and designed by God, or whether there is no God at all. All of this kind of thinking is better left to mature minds, in my opinion, mostly because PARENTS are so protective of what goes into their children's ears (which is not a bad thing).
 
Posted by Libbie (Member # 9529) on :
 
quote:
(though I think there are peaceful ways to compare quality/truth of religions).
Do you? I'd love to hear what you believe would work! [Smile] In my mind, there is no way to "compare" the "quality" or the "truth" of religions. The very phrasing you use there would spark the ire of any fundamentalist of any religion, would it not?

Very many religious people, and particularly those who feel strongly enough about it to wage wars over it, are COMPLETELY CONVINCED that their view is the most quality and the only true one.

I'd think a comparison of whose deeply personal beliefs are the most quality and true would only cause WORSE wars.

I'm not trying to be facetious with this reply - I really would like to know your ideas, and I think your point of view is interesting.

quote:

You seem to have an assumption that when science and religion disagree, choose science (hence, you complaint about "rejecting scientific reasoning for more basic tenets".

As I came up against this very problem in my life and chose science (you could say), I can say that in my case, I chose science because it was logical. Why choose to believe in something illogical when modern science can explain the answers to my questions? Plus, science is testable. God is forever UNtestable, and I don't like that. I happen to think that if there were a God, he'd reveal himself in more obvious ways and that he WOULD be testable. After all, he supposedly loves humanity and *also* desires humanity's explicit worship. Why not demand it in a way that makes his presence totally clear? ...Unless he doesn't have that power, after all, and in that case, why bother to worship Him?

It is also logical that if God exists and gives all good things to men, then he gives scientific knowlege to them because he wants us to know these things and wants us to do good with this knowledge. Yet scientific processes often seem to reveal a universe that does not rely on an intelligent being to keep it ticking.

quote:
Why is the "truth" of science elevated higher than the "truth" of religion?
Whose religion? This is the problem with religion: There are a hundred different variations, some of them WIDELY different from the others, and EVER SINGLE ONE is "the truth." If a Pagan animist wanted you to take their pantheon and their season-worship as truth because they believe it to be truth, how would you feel about that? I assure you, they are every bit as devout in their beliefs and love their gods and sacred texts every bit as much as you do! [Smile] Who is right? Who has the correct version of "truth?" And most importantly, *how do you know?*

With science, there may be doubts and variations on theories, but at least any scientist is free to set up a range of experiments and *test* and *observe* those theories in action, and come to a logical conclusion about which is the truth. With religion, there is no such opportunity - at least, not at present, and not ever in the past. For somebody like me, that answers the entire question right there!

Besides; this question assumes that every person is religious or is able to reconcile some kind of belief in a deity of some sort. Many people in this world do not hold *any* religious beliefs, and have no desire to do so. Why is your take on the world - a take that involves the planning or interference of a conscious deity - truer than mine? [Smile]

quote:
Does science even reveal truth? Or does it just find the best model of given assumptions (which could, of course, be wrong - that's how new theories arise, most often)?
I suppose that depends on how you look at "truth." A very popular theory that has developed out of a bunch of assumptions and models and tests is gravity. But it's still "just" a theory. Do you believe that gravity is the truth? Why or why not? [Smile]

quote:
(What I'm really after is an answer to the last 2 questions I asked, as well as WHY you think that way)
I hope I've explained that adequately for you, even though I have no idea whether your questions were directed at one particular person or the forum in general. Anyway, thanks for a good discussion topic!
 
Posted by Mathematician (Member # 9586) on :
 
So far, more or less *edit* thanks Libbie! */edit* every post can be summarized as follows:

ID has no predictive power, it's not science. Further, we choose science over religion because it's more logical.

I'm trying to get you to question your assumptions, and so far, everyone has simply repeated their assumptions.

I know most (all?) rational people think that ID fails to make a falsifiable claim (if it had predictive power, it would be falsifiable, so the "no predictive power" argument is simply swept up in the bigger "not falsifiable" claim)

Question 1: Why do people assert that it will NEVER make a falsfiable claim? Is there a *logical* reason for that? Or is it simply, "I can't think of a falsifiable claim ID can make, therefore it doesn't have one." If there's not a *logical* reason for it, why do you believe it in favor of the MORE logical, "ID may be able to make a falsifiable claim"?


Further, I KNOW many (most?) people choose rationalism over faith, at least when forced to (i.e., when one's rational thinking and faith contradict). I know science is more logical (almost by definition). But this isn't a reason to choose science over religion. In otherwords, stating we should choose something (science) based on a property(logic/rationalism) it has is foolish in and of itself. There must be a reason WHY that property is more desirable.

Question 2: what makes that property more desirable? Why do we value logic over faith?

As far as the "Do you take reasoning and logic, or do you take a simple, that'll do catch all to shut up all the intelligent questioning", I take the the most accurate description of reality. If that can be discovered by science, wonderful. If not, I'm willing to have it provided by another source.
 
Posted by Libbie (Member # 9529) on :
 
quote:
Originally posted by Amka:
[QB] About ID:

I'm just jumping back in here and haven't been in the pluto thread, so forgive me if I get at all redundant.


Bravo on your entire post, Amka, although I happen to believe that science *can* explain your basic questions you presented. That's just my interpretation of the questions and the way they'd be explained, though. [Wink]
 
Posted by Tresopax (Member # 1063) on :
 
Science means a lot of different things to different people. However, I think the sort of "Science" we should focus on is the sort of science that has a special authority - that we can trust as being true. For instance, a bunch of scientists may get together and declare that a fetus is not a yet a person, and to some this may be "science" speaking, but that does not mean we should drop our own opinions on the issue just because they say it. Such a declaration does not have that special authority, because it is really just a bunch of opinions that happen to be related to science. However, other scientific declarations DO have the authority to make us give up any beliefs to the contrary. For instance, if scientists observe that the moon is not made of green cheese, we should accept that truth. If experiments show that heavy objects fall the same speed as light objects, we should accept that truth. It is the latter category that I think we should call Science - the things that have a special authority, that force us to accept them as being true no matter how much we may not want it to be true.

I think this category of evidence is limited only to (1) that which we can observe in objective, repeatable, measureable experiments, and (2) anything which directly follows from those observations. And the reason this has a special authority is because it is really just observation. If we doubt it, we can try it for ourselves. And if we still doubt it afterwards, we can try it again and again. If the results are always the same, you must accept it, or deny your senses. This is the sort of "Science" that I think is rock solid. You cannot dispute it, as long as the experiments continue to support it. And if your religion disagrees with it, you should probably reject your religion - because that would mean you religion conflicts with what you can repeatably observe to be true.

I'd contrast this to a whole bunch of other sorts of conclusions that may be related to or based in part on scientific results, but are also based on other assumptions. These are extrapolations or extensions, where people take their own biases and opinions, and look at scientific results through that lens. Instead of saying what was observed, these conclusions often discuss what might occur in the future, what certain evidence means to us, or how we should act. Medicine is a prominent example - it tells us how we should act, given what science tells us. The science part of it is based in real observations, but the "should" part of it is based in non-scientific assumptions about right and wrong, such as "it is good to live longer at the expense of not eating Big Macs." While many people put this under the umbrella of science, I think it is dangerous to do so, because there is no reason to give these conclusions the same special authority that direct scientific observations have. A reasonable person should NOT disagree with the evidence that the earth is flat, given that he can repeatably observet that fact over and over if he wants. But a reasonable person CAN disagree with the evidence that they should not eat three Big Macs a day - because while they can definitely observe that doing such a thing would result in less healthiness, there is no way to scientifically show that the value of that health is greater than the value of getting to eat those Big Macs. Science cannot measure value. Thus it would be a mistake to give "Do not eat Big Macs" the same weight of scientific authority that we give to "the world is round". They are in different categories.

So, to answer the question, I think Science consists of whatever we can observe through the scientific method, and the things we can directly conclude from those observations, without requiring other non-scientific assumptions. Everything else may still be relate to science, and may be something scientists and science students should be concerned with, but I would not call it science, strictly speaking.

Frankly, I think making such a distinction more clear would dissolve the need for I.D. After all, I think the real reason it has been proposed is that some groups are worried there is no God in the scientific models. But the real truth is, the parts of the scientific models that address whether or not there is a God involved are totally nonscientific assumptions. They are based on things like Occam's Razor, which is a nonscientific assumption. And thus Science in no way should be taken as saying God doesn't exist. If religious groups and others remembered this, I think they would be less threatened by science. But as it is now, science is lumped in with all sorts of nonscientific ideas, like Occam's Razor - until it has become a sort of religion. It is this, the religion of science, that scares fundamentalists. And it bothers me too, because science should not be a religion. It should be a well-defined, rock-solid set of observable conclusions - separate from the less-solid extrapolations we could derive from those conclusions.
 
Posted by Orincoro (Member # 8854) on :
 
To the thread in general- How bouts we don't take eleven of my words completely out of their original context and argue about them? One way or the other, and I haven't read the thread yet so I dunno, I don't appreciate being called out in a seperate thread that I may or may not notice for a day or two (im a busy starving student after all). If you wanna critique me personally, do it where I can see it and not look like a jerk for ignoring you. [Big Grin]
 
Posted by Orincoro (Member # 8854) on :
 
quote:
Originally posted by Mathematician:


However, as an example, string theory still hasn't made a falsifiable claim. By similar reasoning as above, this currently disqualifies it as being science. Yet, I'm ok with this being taught in a science classroom.

What's the difference?

The difference is that at the moment, there's no hope of ID coming out with scientifically testiable claims. In direct contrast, string theory seems much more likely to begin making these claims.

What this means is that while things like ID may have no current basis in science, that, to me, isn't an arguement for whether or not it should be taught as truth (I'm not saying whether or not it should be, because I haved no idea whether or not ID is true), though I think it SHOULDN'T be taught as science.

So, what do you think? Is science truth? Or is science the best rational approximation to truth? Or something in between? Or something more?

*EDIT* - just to be clear, I do NOT want an ID vs evolution debate, though I suppose I have NO idea where the thread will actually run

Ok I see where you are trying to have a discussion here.

Science, I think, does not attempt to merely supply us with the most convenient or workable answers to practical questions. There is a large practical aspect to science, but that is not the end all. The original formulators of science had to distinguish between logical deduction and falsifiable claims which are proved or disproved by experiment. Early scientists realized that a closer knowledge of the world could be attained by asserting human influence and experiment into an observation in order to understand a process.

For example, Greeks at one time believed that flies formed spontaneously from food, but early scientists, working under the early principles of scientific method in the ionian tradition, covered food with cloth and observed that flies where not able to land on it. Further experiments confirmed that flies lay eggs on food as it rots, thus: more flies.

ID is the conjecture, that flies arise spontaneously from food. It sidesteps science by claiming, as part of its proof no less, that it is unverifiable by the scientific method. It claims to be working within the scientific method, but in its present formulation, it is divorced completely from science. M theorists look VERY hard for ways of falsifying string theory, such as the measurement of gravitational forces for fast moving objects, believing that they may one day prove or disprove the existance of their 11 dimensional model.

In our scientific tradition, ID is not a theory. Evolution is a theory, and that is not "just a theory," it is also an established fact. Scientific thought now takes the theory of evolution through natural selection as a postulate, not provable in the same way that 2+2=4, but as reliable as fact. The day that postulate is outmoded, though it likely never will be, it will be revised and that will be in the best interest of science.

ID cannot be science because ID is not interested in falsifying a claim. ID is VERY interested in falsifying all falsifications of its own claims. ID is interested in obfuscation, misdirection, and a religious, creationist agenda. Science is disinterested in the outcome of experimentation because the outcome of the experiment is a confirmation either of the increased validity, or falsehood of a claim (in my usage, disinterested is to mean "not directly invested"). Either way in experimentation, science always wins, and it can never lie (people do, but not science as a whole, or at least never for long).

ID itself is a tenuous attempt by a religious group to confuse the definitions of "fact," "theory," and truth. Mainly they wish to equate a scientific theory with a complete philosophy which is inimical to scientific thought. A complete philosophy, like for instance Christianity as it has existed since the time of Augustine (see "On Christian Doctrine" for the Platonic influence on Christian thought), is one which claims itself to be self-evident. Christianity and by extension ID, claims that its existance is proof of its validity, and that its inherent validity is justification for faith in all its tenets. This is not intended to be a Christian bashing rant, because I got that information from a central Christian writer who most Christians do not read, Augustine of Hippo.

I mean what i said, in the context in which I said it, that ID is COMPLETELY outside the realm of science. It is not within the realm of science because people say it is. It is not in the realm of science because people want it to be. And it is not in the realm of science, most importantly, because it is about as like science as Bible study. Study the bible if you want, believe in it, believe in intelligent design, but do not couch a religious doctrine as a scientific principle, and demand that people listen simply because you say it is valid. It is not valid.

The day an ID argument puts itself on the line and offers a falsifiable claim, then it will be at least related to the scientific process, and it will likely be proved wrong if the experiment is practical. But here's the thing about that: ID is never going to offer a falsifiable claim because that is inimical to ID. Theories are theories because they are both strong and not overstated. They are simple ideas, which are made into powerful principles over time. ID is a top-heavy, over-stated and completely un-maneagable claim, and there's the kicker- it can't be proved wrong, ever; so it isn't science. If you can't get a chance at proving something wrong, then you can't be sure it isn't completely divorced from reality, a figment of the imagination.
 
Posted by Orincoro (Member # 8854) on :
 
quote:
Originally posted by Mathematician:

Question 1: Why do people assert that it will NEVER make a falsfiable claim? Is there a *logical* reason for that? Or is it simply, "I can't think of a falsifiable claim ID can make, therefore it doesn't have one." If there's not a *logical* reason for it, why do you believe it in favor of the MORE logical, "ID may be able to make a falsifiable claim"?


You are, I presume, a mathematician, and probably aware of the paradox of the unnexpected hanging?

A judge tells a condemned man his sentence is death. There are two stipulations

1. You will die this week, by saturday.
2. You will not know when you wake on that day, that it is the day that you will die.

The man, deduces from this that he cannot die. First of all, if he wakes up on saturday, and he has not yet died, then he must die that day, but he will know that is going to die, thus he cannot die saturday. The same logic applies to all the preceding days, and he confidently rests knowing he cannot possibly die.

He awakes on saturday morning, and the balif comes to his house and he is hanged.

Why did he die? :Spoilers: Because he followed a logical train of thought based on a pare of contradictory claims. The judge was in error, as his two claims were in conflict, and therefore neither of them is valid. He dies anyway, but it is an unpredictable event.

One may see that you can go backwards and claim that because the man does NOT know that he is fated to die on saturday, the stipulations of the judge turned out to be correct. However they are not correct, and their coincidence with what happens is random.

ID, in this way, makes claims which appear to agree with the truth, but do so only because they are constructed to appear so. They are neither predictive nor useful, nor more true than the judge's contradictory stipulations. They may appear to be correct in the end, but that appearance is coincidental, and occurs because the construction of the theory intentionally thwarts falsification.

If that made any sense- I am not an expert on logic, so I suggest further reading on this paradox for more insight than I can hope to offer.
 
Posted by rivka (Member # 4859) on :
 
quote:
Originally posted by Mathematician:
Further, I KNOW many (most?) people choose rationalism over faith, at least when forced to (i.e., when one's rational thinking and faith contradict). I know science is more logical (almost by definition).

I don't. If science says one thing, and my faith another, I will choose faith. Every time.

However, I don't believe that they actually contradict one another nearly as much as some people think they do. Nor do they need to be reconciled as much as some (other) people think. They are separate spheres, and while they certainly overlap (for instance, I consider studying science a small way to get insight into the mind of God, as it were), they are not all that often in conflict. IMO, natch.

That said, I think ID is absolute nonsense. Not because it claims "God did it" -- I believe that to be 100% true. But because it claims "God did it, and science can prove that He did." I am entirely unconvinced that He works that way. To the contrary, I believe it is an essential part of the world He created that we have the freedom not to believe in Him. And unquestionable scientific proof would deprive us of that essential freedom of choice.

Is it possible that ID would make scientifically falsifiable claims? I think it does. However, to the best of my knowledge, the claims depend on a complete lack of understanding of how statistics works. Not a single one bears much scrutiny.
 
Posted by Orincoro (Member # 8854) on :
 
Rivka, that may be the simpler route to the solution: why the heck would a christian care if the christian faith culd be proved by science? That is essentially what ID tries to do.
 
Posted by rivka (Member # 4859) on :
 
<-- not a Christian

[Wink]
 
Posted by Orincoro (Member # 8854) on :
 
Well- the ID people are Christians right?
 
Posted by Orincoro (Member # 8854) on :
 
quote:
Originally posted by Mathematician:
(This was originally going to be posted in the "Pluto's not a planet anymore" topic, but it just seemed to diverge to far from topic).

(I don't mean to call out Orincoro at all, he/she just had the most recent version of this quote)

He- and don't worry about it! Just put the person you are adressing in the title in the future, or make the quote anonymous, that's my advice. [Wink]
 
Posted by rivka (Member # 4859) on :
 
quote:
Originally posted by Orincoro:
Well- the ID people are Christians right?

Not all of them. I know plenty who are Jewish.
 
Posted by Eduardo St. Elmo (Member # 9566) on :
 
Science and faith are both ways in which man tries to make sense out of the things he experiences.

I agree with Rivka that it is impossible for mankind to prove (or disprove) the existence of God (or Jehovah or Allah or whatever name you choose to give to the being that supposedly created everything)

Religion tends to explain things that have (or had) no other plausible explanation. A long time ago, when loads of things that have now been scientifically analysed were still 'a mystery'. The earlier polytheistic religions personified such phenomenons (love, death, war, weather etc.) giving the people the feeling that they had some level of control over these things; by praying and offering to the god(dess) in question.

The monotheistic religions have simplified this in the sense that there's now only one deity, who is responsible for any and all things that come to pass. Praying to him for guidance can soothe your worries no matter what the nature of your problem.

Science is just doing the same; trying to find explanations. But they deviated from the premise that there absolutely has to be some sort of supernatural being that watches over us.

Personally, I think it really unlikely that there is a god out there somewhere, since the scientific answers appear to me to be far more logical and/or plausible (and therefor more believable). However, I must stress that I'm an agnostic, and I am still willing to admit the possibility of the existence of God.

I have no problem with faith, as long as people don't do horrible things (murder, torture and soforth) because of what they believe.

Peace.
 
Posted by Tresopax (Member # 1063) on :
 
quote:
Evolution is a theory, and that is not "just a theory," it is also an established fact.
This is not correct, unless by "established fact" you just mean whatever you believe to be true - in the way that sometimes Christians say it is a fact that God exists or sometimes Democrats say it is a fact that the Iraq War was wrong. But at a minimum we can say that it is not proven that the theory of evolution is true; it could be wrong - and thus I don't think it should qualify as established fact.

quote:
ID cannot be science because ID is not interested in falsifying a claim.
This is the misleading argument that I originally objected to in the other thread. It is true - yes, ID isn't science (strictly speaking) because it isn't interested in falsifying a claim. But that doesn't imply scientists have no business considering it or discussing it in schools. After all, there are many things that don't involve a falsifiable claim that scientists do discuss on the job. The definition of "planet" is one example. One cannot falsify a definition of something through the scientific method, so the definition of "planet" is (strictly speaking) not science, but that does not mean scientists should be discussing the matter. There is a strong argument that such definitions, though non-scientific, are relevant to science. Similarly, I think that there is a pretty strong argument that Intelligent Design (and the entire issue of whether or not some God influences the world), though non-scientific, is relevant to science. After all if God does (or doesn't) influence the world, then that could majorly impact the way we interpret and apply scientific results.

quote:
Is it possible that ID would make scientifically falsifiable claims?
I think it would be pretty easy for ID to make falsifiable claims. But that would require them to define the nature of the Intelligent Designer and how He acts. For instance, it would have to say things like "If we pray X times, then Y will occur." Or, "in situation A, the Intelligent Designer will do B". Those would be falsifiable.
 
Posted by rivka (Member # 4859) on :
 
quote:
Originally posted by Eduardo St. Elmo:
I have no problem with faith, as long as people don't do horrible things (murder, torture and soforth) because of what they believe.

Oh.

Darn.

Sorry guys, no lynchings today. Be careful when dousing your torches, ok? Don't forget about the potluck tomorrow night!
 
Posted by Tarrsk (Member # 332) on :
 
quote:
Originally posted by Tresopax:
quote:
Evolution is a theory, and that is not "just a theory," it is also an established fact.
This is not correct, unless by "established fact" you just mean whatever you believe to be true - in the way that sometimes Christians say it is a fact that God exists or sometimes Democrats say it is a fact that the Iraq War was wrong. But at a minimum we can say that it is not proven that the theory of evolution is true; it could be wrong - and thus I don't think it should qualify as established fact.
Would you say that the gravity is an established fact? Atomic theory? The germ theory of disease? All of these are also scientific theories, and in each case, one could argue that we don't actually have direct, incontrovertible proof. Yes, even for germ theory- sure, we can see exactly how pathogenesis for a given bacterium occurs using microscopy, but how do you know that the microscopes are reliable? You trust the physicists and engineers who built it, but they're also building off scientific principles inferred from the data. To be precisely accurate, you cannot say that *any* of these are "established facts," but we do as a shorthand, because the evidence in their favor is so extensive and strong. The same goes for the theory of evolution (and if you don't believe *that's* the case, then you really need to go review the literature).

quote:
This is the misleading argument that I originally objected to in the other thread. It is true - yes, ID isn't science (strictly speaking) because it isn't interested in falsifying a claim. But that doesn't imply scientists have no business considering it or discussing it in schools. After all, there are many things that don't involve a falsifiable claim that scientists do discuss on the job. The definition of "planet" is one example. One cannot falsify a definition of something through the scientific method, so the definition of "planet" is (strictly speaking) not science, but that does not mean scientists should be discussing the matter. There is a strong argument that such definitions, though non-scientific, are relevant to science. Similarly, I think that there is a pretty strong argument that Intelligent Design (and the entire issue of whether or not some God influences the world), though non-scientific, is relevant to science. After all if God does (or doesn't) influence the world, then that could majorly impact the way we interpret and apply scientific results.
*sigh* And we already told you in the other thread that terminology IS scientific, even if it's technically "not falsifiable." The word "planet" doesn't have to be falsifiable because it doesn't make ANY claims about reality at all. It's just a word with a definition reached by consensus, chosen to best facilitate communication, not for any other reason. Raw data isn't falsifiable either, but it's certainly scientific, because it too does not make any claims beyond "this is what we observed." The data itself does not suggest a conclusion; that is for the researcher to infer BASED on the data (and those conclusions, incidentally, must be falsifiable). ID, on the other hand, does present a claim- that an intelligent designer is responsible for the creation of life. It is claims that must pass the falsifiability criterion in order to be considered scientifically valid. Note that "scientifically valid" is not the same thing as "accurate"- the geocentric model of the world was scientifically valid, because it was possible to disprove it. That it was actually disproved just means that it was incorrect. ID, on the other hand, doesn't even enter the realm of science because there is no way to disprove it at all.

quote:
quote:
Is it possible that ID would make scientifically falsifiable claims?
I think it would be pretty easy for ID to make falsifiable claims. But that would require them to define the nature of the Intelligent Designer and how He acts. For instance, it would have to say things like "If we pray X times, then Y will occur." Or, "in situation A, the Intelligent Designer will do B". Those would be falsifiable.
I think I could accept those hypotheses as falsifiable, but they are based on the assumptions that (a) an Intelligent Designer will actually respond in a predictable, repeatable fashion, and (b) that we can actually come to some sort of agreement about what X, Y, A, and B are. Good luck with that one. In any case, proponents of Intelligent Design have NOT proposed any such falsifiable hypotheses, instead sticking to the tired argument that "if we can't demonstrate exactly how every tiny step of ______ occurs, then the entire theory MUST be wrong." Because, y'know, there are gaps in human knowledge of every single scientific theory we use. There is absolutely nothing we understand down to the most minute detail. Does that make any of the theories I mentioned at the beginning of my post invalid or wrong? What about basic physics? Chemistry? Meteorology? Geology? If we accept the ID argument against evolution, all of those disciplines go out the window.
 
Posted by KarlEd (Member # 571) on :
 
quote:
As far as the "Do you take reasoning and logic, or do you take a simple, that'll do catch all to shut up all the intelligent questioning", I take the the most accurate description of reality. If that can be discovered by science, wonderful. If not, I'm willing to have it provided by another source.
Well this begs the whole question of what is "real", which is another debate entirely. If you believe with all your heart you were "touched by God" in answer to some prayer, does that mean it is true? I'm sure for many that is where faith comes in. However, all the faith in the world (IMO) will not make such a thing "True" or "Real" if in fact you were simply chemically or emotionally unbalanced and "God" had nothing to do with what you felt.

The thing about science vs religion is this:

Science can provide specific answers to specific questions, and if you doubt the answers, you can check them out for yourself and subsequently either confirm them for yourself or discover an error and add to scientific knowledge.

In religion, very little (if anything) is actually verifiable by an unbiased party. Almost nothing is repeatable or testable. There is really only one way that you can "find out for yourself" anything at all, and it is couched in such extremely subjective language that basically religious "truth" is indistinguishable from simply what you want to believe (from an outside observer's standpoint).

So, in regards to this question, "Is science the search for truth? Or is it the search for the best rational/observational/experimental explanation at the time (or is there a difference between the two?)" I'll take option number 3. I think reason, observation and experimentation are essential tools in the search for truth. I'm willing to accept that these tools can only give us "provisional truth", and that further observation, experimentation, and reason will likely always (as far as we humans are concerned) yield more precise truths. So in science, there is no difference between the two.

Religion can also be said to be the search for truth, but I'm not sure to what degree it could be said to fit the second definition. Clearly observation and (hopefully) reason play a role in one accepting religious truth, but I'm not sure to what degree experimentation plays a role. That probably differs from person to person, but I'm relatively confident that no religious experiments give replicable results regardless of who is doing the experiment.

Maybe it would be more precise to say that Science is the search for universal truth and Religion is the search for personal truth.
 
Posted by Libbie (Member # 9529) on :
 
quote:
This is not correct, unless by "established fact" you just mean whatever you believe to be true - in the way that sometimes Christians say it is a fact that God exists or sometimes Democrats say it is a fact that the Iraq War was wrong. But at a minimum we can say that it is not proven that the theory of evolution is true; it could be wrong - and thus I don't think it should qualify as established fact.
I could be wrong, but I believe that all of science agrees that evolution happened or is happening (or both). The debate lies in the specific processes of evolution - the specific flavor, if you will. Just as all of science agrees that it is gravity that holds us to the Earth, so gravity's influence is considered to be real. In the same way, evolution is considered to be factual, even if not every scientist agrees on just how it happened or is happening.


quote:
It is true - yes, ID isn't science (strictly speaking) because it isn't interested in falsifying a claim. But that doesn't imply scientists have no business considering it or discussing it in schools.
I have to say, this statement is very confusing to me. If scientists are interested in considering and discussing science, and ID is not science, then why should scientists consider it or discuss it in schools?

What often gets overlooked in this ongoing debate is the rights of the nonreligious children in schools. It's fine to discuss ID in broad, nondenominational terms that will suit the needs of the religious students. What about the atheists? More families are choosing to raise their children without religion all the time. Do these kids not have a right to be free from indoctrination in public schools?

Sorry, that is a tad off the topic, so I'll reel it back in with this thought: If evolution itself is broadly accepted as factual by science (just not the specific processes), then what's wrong with presenting the idea of evolution in schools without any assertion that evolution *must* lack intelligent control or design? When I was learning about evolution in school, I just applied my own beliefs to it and said, "Wow, this sure is interesting. God must control this process according to His big plan! Neat!" Why can't children be encouraged to apply their beliefs to what science considers a fact?

quote:
Similarly, I think that there is a pretty strong argument that Intelligent Design (and the entire issue of whether or not some God influences the world), though non-scientific, is relevant to science. After all if God does (or doesn't) influence the world, then that could majorly impact the way we interpret and apply scientific results.
I see your point here. However, we've already proven the world to be a mighty predictable place, with enough science and math applied and over a large enough sampling of any given variable. Isn't it logical that intelligent control over it would cause almost total unpredictability?

quote:
I think it would be pretty easy for ID to make falsifiable claims. But that would require them to define the nature of the Intelligent Designer and how He acts. For instance, it would have to say things like "If we pray X times, then Y will occur." Or, "in situation A, the Intelligent Designer will do B". Those would be falsifiable.
True - but then, of course, those claims would be put to the test and proven wrong (because as all religious folks know, the Lord works in mysterious ways). And that would end ID's chances of being indoctrinated into public education.

Unless God allowed himself to be proven via scientific process just to stop this debate. [Wink]
 
Posted by pooka (Member # 5003) on :
 
I believe in God, but I grant science is as good of an investment as anything the government spends its money on. I don't know that I've ever proclaimed that I believe in ID, since I associate it not with a scientific theory but with a political movement. I feel the same about "Darwinism". There is no truth in science, only the presentation of evidence that does or does not withstand scrutiny, or which is a fruitful field for further inquiry. In a sense, the laws of natural selection themselves apply to the academic ecology.
 
Posted by Tresopax (Member # 1063) on :
 
quote:
Would you say that the gravity is an established fact? Atomic theory? The germ theory of disease?
I would say they are scientific theories, not established facts - because they each make predictions about the future that could reasonably turn out to be false.

quote:
The word "planet" doesn't have to be falsifiable because it doesn't make ANY claims about reality at all.
"Planet" is just a word. But if you say "the proposed definition of 'planet' is better than the old definition of 'planet'", then that is a claim about reality, and is not falsifiable.

Terms are scientific when being used in the scientific method. But arguing over which definition of terms is better is not scientific - that's semantics.

quote:
Raw data isn't falsifiable either, but it's certainly scientific, because it too does not make any claims beyond "this is what we observed."
Raw data is scientific insofar as it is used in the scientific method to test falsifiable theories. It is not scientific outside that. For instance, I'm getting data as I check the clock right now - but that does not mean I am doing science.

quote:
think I could accept those hypotheses as falsifiable, but they are based on the assumptions that (a) an Intelligent Designer will actually respond in a predictable, repeatable fashion, and (b) that we can actually come to some sort of agreement about what X, Y, A, and B are.
Well, yeah, I doubt they will agree to these - which is probably why they will have a hard time ever making a science out of ID. If they reject (a) then they are rejecting a foundational assumption of science - they'd be arguing that the universe does not operate in a predictable, repeatable fashion. (Which would be a non-scientific argument, although I think it most certainly would be an argument relevant to science.)

That may be the real problem here. Perhaps the argument that religious groups really should be making is that science itself is based on mistaken assumptions. Perhaps they should be arguing that the world is inherently unpredictable.

The irony is that if Intelligent Design succeeded and became scientific, it would probably be the biggest threat to established religion that there is. After all, that would mean we would be able to test the nature of God - and determine what dogma is wrong.
 
Posted by Libbie (Member # 9529) on :
 
Orincoro, Tarrsk, and KarlEd, I really admire your posts here and I hope to meet you and shake your hands one day.
 
Posted by Libbie (Member # 9529) on :
 
quote:
Originally posted by pooka:
I believe in God, but I grant science is as good of an investment as anything the government spends its money on. I don't know that I've ever proclaimed that I believe in ID, since I associate it not with a scientific theory but with a political movement. I feel the same about "Darwinism". There is no truth in science, only the presentation of evidence that does or does not withstand scrutiny, or which is a fruitful field for further inquiry. In a sense, the laws of natural selection themselves apply to the academic ecology.

That's an interesting viewpoint, pooka!

From my point of view, there is no truth in religion because there is *no* presentation of evidence that does or does not withstand scrutiny; and at least science provides testable, observable *understanding,* which I don't find in any religion.

I suppose the entire crux of this problem is how an individual defines truth. [Smile]
 
Posted by Tarrsk (Member # 332) on :
 
quote:
Originally posted by Tresopax:
quote:
Would you say that the gravity is an established fact? Atomic theory? The germ theory of disease?
I would say they are scientific theories, not established facts - because they each make predictions about the future that could reasonably turn out to be false.
Then we're actually in agreement here. To be honest, I was a bit uncomfortable with referring to evolution as "established fact," myself, because of exactly what you say. You cannot prove something to be true- you can only state that, based on current evidence, it has not been proven *false.* In cases like atomic theory, germ theory, and evolutionary theory, though, the current evidence in their favor is so strong that there's no real need to reaffirm their "theoryhood" every time they come up.

quote:
quote:
The word "planet" doesn't have to be falsifiable because it doesn't make ANY claims about reality at all.
"Planet" is just a word. But if you say "the proposed definition of 'planet' is better than the old definition of 'planet'", then that is a claim about reality, and is not falsifiable.
No, it's not a claim about reality. In this case, the use of the word "better" is misleading. The new definition of "planet" is not "better" at explaining anything about the nature of reality. The only thing it's "better" at than the old definition (and even this is debateable) is that it more clearly delineates between orbiting bodies that share some similarities and Kuiper bodies. But it doesn't say anything about *why* or *how* these bodies have similarities; nor does it make any claims that these similarites are, in fact, at all important in the grand scheme of things. The definition exists only because scientists find it a more convenient one to use. It is most emphatically not a claim about some universal truth- an alien culture could happily use some other definition of "planet" and both our scientists and theirs would be perfectly justified.

Let me use another example, one from biology. The term "species" is commonly defined as "a group of organisms that can interbreed to form viable offspring." However, in reality, life is far more fluid than this simple definition. Evolution being what it is, you can have a large population of organisms in which the group on one side of the populated range is readily distinguishable from the group on the other side, but each group can breed perfectly well with the groups immediately surrounding it. In one particular instance (a bird in the Western United States, I believe), it was demonstrated that even though each subpopulation of this bird could breed with their immediate neighbors, a bird from the most northern borders of the range and a bird from the south cannot. In this case, how do you define which birds should belong in the same "species"? Ultimately, you have to realize that it doesn't matter at all, because the term "species" is a human construct that we use as a matter of convenience. Evolution doesn't deal in discrete kinds, but instead in genetic continuums of populations. Does this make the word "species" non-scientific? No, because the fact of the matter is, in most situations, species are isolated enough that the traditional definition works just fine. We simply acknowledge that our definition is flawed, and make note of the problems whenever it is necessary.

quote:
quote:
Raw data isn't falsifiable either, but it's certainly scientific, because it too does not make any claims beyond "this is what we observed."
Raw data is scientific insofar as it is used in the scientific method to test falsifiable theories. It is not scientific outside that. For instance, I'm getting data as I check the clock right now - but that does not mean I am doing science.
I think we're talking past each other here. I am saying that data is scientific in the sense that it has a role in scientific inquiry. And furthermore, by "data," I am referring specifically to observations taken in a systematic fashion, with appropriate controls. This encompasses both experimental data and observational data, such as information about population biology or geological analysis of a volcano. When I read the luminescence of bacteria expressing beta-galactosidase, I am most definitely "doing science," even if I haven't reached the point where I can say anything definitive about my results yet.

quote:
quote:
think I could accept those hypotheses as falsifiable, but they are based on the assumptions that (a) an Intelligent Designer will actually respond in a predictable, repeatable fashion, and (b) that we can actually come to some sort of agreement about what X, Y, A, and B are.
Well, yeah, I doubt they will agree to these - which is probably why they will have a hard time ever making a science out of ID. If they reject (a) then they are rejecting a foundational assumption of science - they'd be arguing that the universe does not operate in a predictable, repeatable fashion. (Which would be a non-scientific argument, although I think it most certainly would be an argument relevant to science.)

That may be the real problem here. Perhaps the argument that religious groups really should be making is that science itself is based on mistaken assumptions. Perhaps they should be arguing that the world is inherently unpredictable.

They can certainly argue that, and there are certainly epistemologies that do. But they run up against the fact that every aspect of modern human technology is founded upon the assumption that the universe is predictable. It's possible to make an academic case against a predictable universe even under these circumstances, but given the massive success of science so far, there's no reason for anyone to accept it in any practical fashion.
 
Posted by Mathematician (Member # 9586) on :
 
Interesting posts, thank you.

(sorry about the delay in getting back to you...had a 5 hour math test today, spent lots of time studying for it)

I don't want to get into the debate about the role of definitions in science. Clearly, agreed upon definitions are needed for ALL forms of communicating. As a mathematician, I especially understand this.


One thing, though. After a bit of introspection, I found that, at least to me (and I'd bet to most of you as well), there's something inherently more satisfying to an answer provided by science, as opposed to faith. I suppose it's the mathematician in me coming out - I want to understand everything at it's core.

That said, I have another dilemna about the logic vs faith debate.

Many of you have indirectly referenced the scientific method as the best way to test "truth", or "reality", or some other (hopefully) less ambiguous term.

Here's the issue. Can you USE the scientific method to somehow show the scientific method is best?

If not, why accept it to begin with? It seems, with a touch of irony perhaps, that our trust of logic depends on faith. Note that if we can't use the scientific method to prove the scientific method works best, then more or less everyone's reasons for following logic more than faith become worthless. This debate hinges on this one assumption - that logic is somehow better than faith at gleaning truth.

To give my own partial answer, I think it's something (culturally?) ingrained in us. I imagine every response to the question (which actually answers the question) will be an appeal to reason, not to faith. We simply are used to using logic to communicate our views, perhaps for the precision of it, perhaps for some (many?) other reasons.

Unfortunately, ths answer doesn't help answer anything in the faith vs logic debate.

Maybe we just choose logic over faith because it's more ubiquitous and easier (to communicate, to be satisfied with, etc)?

I don't know. Maybe I'm just rambling (yay for 5 hour tests!) I'm gonna let my brain rest now. Again, thanks for all the wonderful responses!


PS (I'll take your advice Orincoro, thanks for understanding!)
 
Posted by King of Men (Member # 6684) on :
 
quote:
Here's the issue. Can you USE the scientific method to somehow show the scientific method is best?
Actually, I think you could. To wit, you would study the correlation between the success of a culture (measured by a purely Darwinian standard of survival versus non-survival; that way you get no issues of what 'success' is defined as, because whatever the definition, no culture which doesn't exist can have it) and its use of the scientific method. I think it's pretty clear that you don't really need to do the study; tanks and machine guns will win over faith and connection with nature every time.
 
Posted by Orincoro (Member # 8854) on :
 
quote:
Originally posted by Tresopax:
quote:
Evolution is a theory, and that is not "just a theory," it is also an established fact.
This is not correct, unless by "established fact" you just mean whatever you believe to be true - in the way that sometimes Christians say it is a fact that God exists or sometimes Democrats say it is a fact that the Iraq War was wrong. But at a minimum we can say that it is not proven that the theory of evolution is true; it could be wrong - and thus I don't think it should qualify as established fact.

Tres, your unwillingness to understand or listen to the scientific definition of these words is causing problems in the conversation.

Let me put an established fact into some relief: The "luminiferous ether" was a scientific fact depended upon by newton when he devised his theories of the nature of light, and of universal gravitation. He used it as the basis of his theories and it worked very well, it agreed with hundreds of years of observation as well as his own calculations. It was fact to Newton and it aided him in envisioning the universe. It does not exist and never did. Einstein's general relativity proved the needlessness of luminiferous ether, as did experiments having to do with the dual nature of light. When we discovered a different nature of the universe, Newton's observations and the ether's usefullness as a concept did NOT become invalid, they became outdated.

My point is to show that questions in science, and FACTS yes FACTS in science, can be proved outdated or innacurate, but they are rarely proved wrong. Newton was not wrong in his calculations, he was ignorant of subtler forces in the universe. Einstein was not WRONG in his theory of general relativity, he may also have been ignorant of yet subtler shades of reality in the 11 dimensional space model. Some theorists believe that the next century will bring proof that Einstein's equations are not accurate in special situations having to do with fast moving objects, but Einstein will not have been proved WRONG, and relativity will still be an established fact. The existance of vacuum energy, if it can be proved, will add a new understanding of the Einstein model which will force an adjustment to our ideas of relativity, but they will not be WRONG.

On the reverse side, ID people like you claim that these theories are never "proved true," as you stated in your post. This is ridiculous. Plain and simple, you have no concept of the difference between ID and the Theory of Evolution by natural selection. The past centuries have seen mountains of evidence collect in favor of modern theories of evolution; this is why those theories are modern, because they explain old discoveries and predict new evidence which is subsequently found. From darwin's day to today, we have added genetics, DNA, radio-carbon dating, and myriad other techniques to our toolkit for understanding evolution; it is no longer Darwinism, but a modern understanding of evolution which is NOT based on a conjecture but on concrete evidence. That evidence is not arranged in such a way as to favor an evolutionary model; more interest is taken by science in those elements of the current model which are inconsistent, as is the way with science.

If evolution has not yet met your standards for being proved "true," though I will stress that scientists don't get a rat's tail about proving it true, then it NEVER will. We cannot prove that pie is a transcendental number by writing it out forever- because it would take FOREVER. We prove it, in a way which is satisfactory to us, by an advanced series of deductions. Pie is a postulate, and I suppose you would argue that this means it is not a fact. In this case, NOTHING is a fact, and we have to teach EVERY possibility to EVERYTHING.

There are not "two sides" to this debate, nor are there "two theories." One is a theory, and the other is a belief that religious zealots and misinformed people would like us to believe is as valid, in a scientific discussion, as science.


As to your mention of the argument over the definiton of planet. This is barely answerable because it is so laughable. Look at the very good answers I and others gave in that thread- I have a hard time believing that you think an argument over a disciplinary jargon makes science less credible as a field. Give me a big break from that Kit Kat bar you've been choking on.
 
Posted by Orincoro (Member # 8854) on :
 
quote:
Originally posted by Tresopax:

quote:
Is it possible that ID would make scientifically falsifiable claims?
I think it would be pretty easy for ID to make falsifiable claims. But that would require them to define the nature of the Intelligent Designer and how He acts. For instance, it would have to say things like "If we pray X times, then Y will occur." Or, "in situation A, the Intelligent Designer will do B". Those would be falsifiable.
And how is the "power of prayer" to be tested scientifically? When prayer is a traditional process evolved out a complex set of social conventions and historical occurences? For instance,the languages we employ in prayer are all different, and are affected by history, which is why we include gramatical and vocabulary usages which are not in common use, only in prayer. All these variables would be impossible to control- and a control group would be impossible to construct, because prayer is not a thing to be measured, it is a vague concept which is different for different people and cultures.

ID preserves itself this way because it claims that the complexity and personal nature of the human relationship with a divine being is untestable. I happen to agree for different reasons, but then we are all in agreement that ID is never going to offer a falsifiable claim- not because it WON'T, but because even if it tried, the falsification of a theory of prayer or divine intervention is untestable. When you are talking about something that is specifically believed to conform to laws that we explicity cannot know, and which is NOT tied to the laws we DO have experience with, the "theory" has absolutely no connection with experimentation or observation.

What we are talking about are things you cannot experience in a common way, for example 2+2=4 is not a metaphysical argument, it is a fact. But "the sky is beautiful today," though you could take a survey and make it look very scientific, is not a scientific claim, it is outside any possible common experience, it deals with too many factors and it is philisophical.

Now I whole heartedly believe in the study of philosophy to understand human thought and our relationship to nature, but I do not pretend that Nietzsche is a scientist when he talks about the "eternally varied harmony of being," or some such philisophical caveat. He doesn't attempt to prove things in a way that can be reproduced by someone else with a completely different personality or even a different language, which is why many still believe that we should read philosophy in its original language, but we make no such arguments about science.

Edit: to Mathematician, yes you are right to say that agreed upon definitions are key in science, that is absolutely vital. I think of how this applies to my argument: agreed upon definitions are NOT necessary in philosophy, and in fact many philosophers center their arguments around the nature of definitional conventions. This practice goes back millenia in philosophical traditions, and it helps to focus the creative energies, and the expressive capabilities of poets, musicians, writers, orators, and scientists. This is part of why ID is not science, it depends on a variable definition of words and concepts according to how they are needed to function, and it is an invitation to philosophical debate, and not to scientific research. The work of ID "theory" is going to be done in the abstract, just as the work of the Christian philosophers and the greeks were done in monasteries and libraries, disconnected from the tangible world, of a purpose, to encourage investigation of the metaphysical and the intangible. This is all fine, but this is clearly not the same thing as science. There are philosophical questions about science, there is a philosophy OF science, but science does not continually redefine its own relationship with the subject, and scientific philosophy does not change the outcome of a repeatable experiment. The philosophy of science is practical, because we already have an impractical philosophy of life, and we call that religion. (again, not bashing, but simply using the appropriate terms: practical vs. impractical has to do with the ability to experiment in this usage).

[ August 29, 2006, 09:18 PM: Message edited by: Orincoro ]
 
Posted by Orincoro (Member # 8854) on :
 
quote:
Originally posted by Mathematician:


If not, why accept it to begin with? It seems, with a touch of irony perhaps, that our trust of logic depends on faith. Note that if we can't use the scientific method to prove the scientific method works best, then more or less everyone's reasons for following logic more than faith become worthless. This debate hinges on this one assumption - that logic is somehow better than faith at gleaning truth.

To give my own partial answer, I think it's something (culturally?) ingrained in us. I imagine every response to the question (which actually answers the question) will be an appeal to reason, not to faith. We simply are used to using logic to communicate our views, perhaps for the precision of it, perhaps for some (many?) other reasons.

Sorry for the triple post- this thought simply leads me to ask why we don't all become Nhialists. (sp?)

If you want to know what 2+2=, then you have to trust that 2 is 2 and that is all there is to it (no pun intended!). If you go down that road- what if two isn't really two man? Then you're just begging the listener not to hear anything you say, because you've reached down into the core of it all and just said, trust ABSOLUTELY nothing. In this case science and religion and EVERYTHING is useless- so what's the point?
 
Posted by Palliard (Member # 8109) on :
 
This is more broadly known in philosophic circles as the "Evil Deceiver" argument, and is the one that so vexed Descartes that he was reduced to one first principle: "I think, therefore I am".

To be fair, there are those philosophers that believe he leaped to an unwarranted conclusion, and that his statement of self would more properly read something like "I think I think, therefore I think I am."
 
Posted by TomDavidson (Member # 124) on :
 
quote:
Why do people assert that it will NEVER make a falsfiable claim?
For my part, it's because even the most "speculative" claims of ID, if true, actually wind up being another form of evolutionary pressure. If God's out there tinkering with genes, that doesn't disprove the theory of natural selection any more than our ability to breed dogs for long tails does. The only thing that "breaks" this argument is that if God is deliberately and randomly interfering with what appear to us to be scientific mechanisms to the extent that the mechanisms themselves do not function in the way they appear to function -- and if that's the case, then observational science is impossible anyway and falsification is moot.
 
Posted by Dagonee (Member # 5818) on :
 
I'm going all the way back to the beginning:

quote:
Is science truth? Or is science the best rational approximation to truth? Or something in between?
Science is not true, not is it the best rational approximation of truth. Rather, science is the best means for discovering certain kinds of truth about the world. It is not the only means of discovering truth, nor is it suitable for some of the most important truths we need to discover.

For example, science can never tell us what we should do. It can tell us what we should do if we desire a particular outcome. But the selection of which outcomes are desired ultimately depend on something science cannot apply to.

An outcome is either desirable because we know it will lead to some other outcome we find desirable or because it is desirable in and of itself. Science cannot define the outcomes in the second group.

For example, science can tell us that if we rotate crops, we will produce more food. Science can also tell us that if we don't produce more food, X number of people will die who would have otherwise lived.

Science does not tell us whether the outcome of X people dying is better or worse than the outcome of X people not dying.

Sure, you could come up with scientific predictions that the death of X people will cause the population to dip below the sustainable level for a given group, leading to that group's extinction. But then we've simply moved to a new outcome that is called good or bad based on a non-scientific principle.

This is not a criticism of science. It is a recognition of its place in the world.
 
Posted by TomDavidson (Member # 124) on :
 
quote:

Science does not tell us whether the outcome of X people dying is better or worse than the outcome of X people not dying.

Specifically, science cannot define the word "better." Science isn't really in the business of definitions at all, now that I think of it.

Science can take things to which you've applied definitions and weigh their effectiveness, but the actual definitions themselves have to come from premises that are at root culturally axiomatic.
 
Posted by The Rabbit (Member # 671) on :
 
There is an enormous problem with comparing intelligent design to super string theory. Super string theory has a rigorous mathematical definition, where as ID has at best a very ambiguous definition. So when we are talking about super string theory, it is at least very clearly defined what we are talking about. That kind of rigorous definition is the first essential step in creation a scientifically falsifiable hypothesis.

Before we can even begin to discuss whether ID has the potential to ever generate a scientifically falsifiable hypothesis, we have to agree on some sort of rigorous definition of ID and that is non trivial.

Consider the following description of intelligent design from the center for science and culture.

quote:
The theory of intelligent design holds that certain features of the universe and of living things are best explained by an intelligent cause, not an undirected process such as natural selection.
To generate a scientifically falsifiable hypothesis from ID theory, we would have to identify not what types of things are commonly caused by intelligence but we would have to identify features of a universe which could not be caused by an intelligent being.

Most of the dictionary definitions of intelligence or intelligent are very circular but consider for a moment the following definition.

Intelligent: possessing the ability to act in a purposeful manner in order to obtain a preconceived result.

Given what we know about intelligent beings (assuming that humans are an example of intelligent beings), intelligent beings are capable of acting randomly and without purpose. So although intelligent beings are capable of applying knowledge to in a purposeful manner, they often don't. That means that there can not be features of the Universe which could have been caused by random processes but which could not have been caused by an intelligent designer. This makes it impossible to develop a postulate which could disprove ID theory.

Furthermore, before we could begin to postulate something based on ID, we would have to dramatically narrow its scope. Which features of the Universe are we talking about? How do we quantify "best explained".

Whats more, ID theory does not specify that all features of the universe were intelligently designed. That means that even if we could identify some feature of the Universe which could not have been designed by an intelligent being, it could never preclude the possibility that some other feature(s) of the Universe was designed by an intelligent being.

This is why I disagree fundamentally with Mathematicians postulate that ID theory could concievably at some future point produce a scientifically falsifiable claim. The theory, in its current use, is simply so imprecisely defined that it can never produce a testible hypothesis.
 
Posted by Orincoro (Member # 8854) on :
 
That is quite interesting rabit, I hadn't considered the nature of intelligence in ID until now.

This reminds me of one of my favorite books, CONTACT, by Carl Sagan, in which he actually alludes to intelligent design, but in a totally other context from evolution.

:Spoilers, though its a good book so read it anyway!:

In the novel, Ellie meets a representative of an intergalactic community of highly intelligent races who tells her about a race of beings who have dissapeared from the universe, but who were so powerful that they could subtly change the nature of mathematical values and physical laws. For instance, the alien alludes to the presence of messages contained within the value for Pi, as well as many other constants of the universe. The alien suggests that this superpowerful race constructed an intergalactic system of communication and transportation which still exists, and that this race redesigned the universe to allow it to continue on after they were gone.

This is an alien race which believes in an intelligent design based on falsifiable claims, which they are attempting to investigate, that an intelligent race, billions of years in the past, redesigned the universe. Pretty neat idea!

:end of spoilers:

It occurs to me that this ID argument is so flawed mainly because it springs from a religious movement with all the wrong intentions in trying to turn the science classroom into a religious front (mainly then, political and not academic or scientific reasons). A theory of ID, in its proper context, would have to be the result of a long and exaustive search for raw evidence that would probably last millenia, and scour the known and unknown universe for possible clues, including the existance or non-existance of other races, and their ideas about these concepts. The problem with ID today is that it springs from a completed philosophy, and not from a searching impulse or a need to explore. Quite the opposite, ID wants to shut down discussion and research by explaining things using the terminology of the inexplicable. This is unnacceptable to me, not because I think ID is not something people may come to believe in 100,000 years, but because it is only really trying to stop people from learning things, and growing in knowledge- it will turn the search for knowledge into the search for wording, or faith, and those searches are already going on elsewhere, where they belong.
 
Posted by rivka (Member # 4859) on :
 
Dags, very well put. [Smile]
 
Posted by orlox (Member # 2392) on :
 
Is natural selection a scientifically falsifiable hypothesis?
 
Posted by Palliard (Member # 8109) on :
 
quote:
Is natural selection a scientifically falsifiable hypothesis?
In a sense, it is, if you can find that something that completely violates the hypothesis: a billion-year-old human fossil, the remains of an angel... stuff like that.

That's one of the reasons the Paluxi Tracks were such a big to the "Creation Science" folks. Turns out they were dinosaur tracks after all, but the search does continue for the "deal-breaker" in evolutionary theory, something so absurd that you'd just have to junk the theory entirely in order to explain it.
 
Posted by Tresopax (Member # 1063) on :
 
quote:
My point is to show that questions in science, and FACTS yes FACTS in science, can be proved outdated or innacurate, but they are rarely proved wrong. Newton was not wrong in his calculations, he was ignorant of subtler forces in the universe. Einstein was not WRONG in his theory of general relativity, he may also have been ignorant of yet subtler shades of reality in the 11 dimensional space model. Some theorists believe that the next century will bring proof that Einstein's equations are not accurate in special situations having to do with fast moving objects, but Einstein will not have been proved WRONG, and relativity will still be an established fact. The existance of vacuum energy, if it can be proved, will add a new understanding of the Einstein model which will force an adjustment to our ideas of relativity, but they will not be WRONG.
I disagree. I think that if a model fails to hold true in any instance, then it is wrong (not true). It may be a useful close approximation, but close approximations are still wrong. Einstein was wrong - but he was close. And thus his theories are not fact - they are just useful theories.

quote:
Edit: to Mathematician, yes you are right to say that agreed upon definitions are key in science, that is absolutely vital. I think of how this applies to my argument: agreed upon definitions are NOT necessary in philosophy, and in fact many philosophers center their arguments around the nature of definitional conventions.
This is not true. Agreed upon definitions are essential in philosophy - probably more so than any other discipline except math. That is why philosophers spend so much time arguing over definitions. If they can't generate agreement, they can't move forward - and often they don't.

I also believe, though, that in philosophy (and in science, which is a specialized part of philosophy) it is also important to have CORRECT definitions. They must be agreed upon AND correct. Otherwise, we could agree that Republican means "supported the Iraq War" and conclude that OSC is a Republican, when he isn't. For this reason, I don't think it is usually productive to simply agree upon a popular definition for something, if it is not correct.

quote:
On the reverse side, ID people like you claim that these theories are never "proved true," as you stated in your post.
I'm not sure what you mean by "ID people" but please note that I am not a believer in the Intelligent Design theory.

Also, people have been saying scientific theories are never proved true long before Intelligent Design was ever around. Karl Popper made a major point of it when he proposed his falsification criteria for defining science. I simply agree with his argument.
 
Posted by Orincoro (Member # 8854) on :
 
quote:
Originally posted by Tresopax:


quote:
Edit: to Mathematician, yes you are right to say that agreed upon definitions are key in science, that is absolutely vital. I think of how this applies to my argument: agreed upon definitions are NOT necessary in philosophy, and in fact many philosophers center their arguments around the nature of definitional conventions.
This is not true. Agreed upon definitions are essential in philosophy - probably more so than any other discipline except math. That is why philosophers spend so much time arguing over definitions. If they can't generate agreement, they can't move forward - and often they don't.

I also believe, though, that in philosophy (and in science, which is a specialized part of philosophy) it is also important to have CORRECT definitions. They must be agreed upon AND correct. Otherwise, we could agree that Republican means "supported the Iraq War" and conclude that OSC is a Republican, when he isn't. For this reason, I don't think it is usually productive to simply agree upon a popular definition for something, if it is not correct.

[ROFL] [ROFL] [ROFL]

If you believe that philosophers rely on a common set of definitions you are wrong wrong, oh so very wrong. Read a little Emmanuel Kant, Augustine, Nietzsche, every one of them spends terrific energry speaking to the universal application of certain words, and trying to define them: beautiful, agreeable, good, true, all have a different view.

Not that I expected you to actually know anything about this, much less understand anything you didn't already think you knew. Oh well. I suppose my definitions of "intelligent debate" and "knowledge" are different from yours. But I guess everyone is entitled to his own opinion, even really stupid ones. Forgive me for being rude, its hard to think of anything nice to say.
 
Posted by Mathematician (Member # 9586) on :
 
quote:
Originally posted by Orincoro:
quote:
Originally posted by Mathematician:


If not, why accept it to begin with? It seems, with a touch of irony perhaps, that our trust of logic depends on faith. Note that if we can't use the scientific method to prove the scientific method works best, then more or less everyone's reasons for following logic more than faith become worthless. This debate hinges on this one assumption - that logic is somehow better than faith at gleaning truth.

To give my own partial answer, I think it's something (culturally?) ingrained in us. I imagine every response to the question (which actually answers the question) will be an appeal to reason, not to faith. We simply are used to using logic to communicate our views, perhaps for the precision of it, perhaps for some (many?) other reasons.

Sorry for the triple post- this thought simply leads me to ask why we don't all become Nhialists. (sp?)

If you want to know what 2+2=, then you have to trust that 2 is 2 and that is all there is to it (no pun intended!). If you go down that road- what if two isn't really two man? Then you're just begging the listener not to hear anything you say, because you've reached down into the core of it all and just said, trust ABSOLUTELY nothing. In this case science and religion and EVERYTHING is useless- so what's the point?

I certainly wasn't suggesting nihilism. I was suggesting that faith and logic are intertwined in a fundamental way. Sure, logic is great, but to get things started in any sort of meaningful way, we need faith (faith that we are not just brains in a vat, for example).

As far as your specific example, that ones a bit easier. First, mathematicians have an agreed upon definition of 2. Likewise, they have a definition of 4. There is no ambiguity in asking what 2 is because we've nailed it down - said THIS THING is 2 and nothing else is.

Second, mathematics is based on logic. While formal logic has a firm foundational setting, it is still inescapably tied with "naive" or "intuitive" logic. But, mathematics recognizes this....

In the same way that mathematics has its own axioms, logic does as well. Things like "for any statement P, (P or P) implies P", and other "self-evident" statements. The way all mathematics works is really with an implied "assuming these axioms of mathematics and these axioms for allowable deductions..."

Thus, 2+2 = 4 not because of some lucky definition, but because we're starting (with faith, if you will), with a collection of "allowable starting points (math axioms)" as well as "rules of deduction I can apply to any mathematical statement (logic axioms)". Then 2+2 = 4 is a neccesary consequence of following the allowable logical deductions.

But the thing is, the axioms, and even the foundational logic, are faith based at the core. We're blindly accepting these statements as being true, just so we can make some progress. In fact, some fields of math/logic look at what happens if we replace some of these "usual" axioms with others, and the results are often quite mind-bending.
 
Posted by Dagonee (Member # 5818) on :
 
quote:
If you believe that philosophers rely on a common set of definitions you are wrong wrong, oh so very wrong. Read a little Emmanuel Kant, Augustine, Nietzsche, every one of them spends terrific energry speaking to the universal application of certain words, and trying to define them: beautiful, agreeable, good, true, all have a different view.
And the reason they spend so much energy on defining them is because "agreed upon definitions are essential in philosophy."

Tres didn't say that philosophers agree on definitions. He said that agreement is essential.

quote:
Not that I expected you to actually know anything about this, much less understand anything you didn't already think you knew. Oh well. I suppose my definitions of "intelligent debate" and "knowledge" are different from yours. But I guess everyone is entitled to his own opinion, even really stupid ones. Forgive me for being rude, its hard to think of anything nice to say.
Then try silence. Especially when your rudeness exposes your ignorance of the subject.

How on earth do you think citing philosophers spending terrific energy trying to define terms somehow refutes what Tres said, when he himself commented on the how much energy they spend trying to define terms.
 
Posted by John Van Pelt (Member # 5767) on :
 
quote:
Rivka wrote:
But because [ID] claims "God did it, and science can prove that He did"

This isn't quite accurate. ID claims, "(a) Something purposeful (OK, call it God) did it, (b) science hasn't proven that it/he didn't, (c) many of the gaps/unknowns/conundrums in current evolutionary knowledge cannot (ever) be solved within science, and thus explanations invoking a designer must be invoked, and (d) the claim presented in (c) requires no scientific proof because it is patently self-obvious."
quote:
Tresopax wrote:
I think the sort of "Science" we should focus on is the sort of science that has a special authority - that we can trust as being true. For instance, a bunch of scientists may get together and declare that a fetus is not a yet a person, and to some this may be "science" speaking, but that does not mean we should drop our own opinions on the issue just because they say it. Such a declaration does not have that special authority, because it is really just a bunch of opinions that happen to be related to science. However, other scientific declarations DO have the authority to make us give up any beliefs to the contrary. For instance, if scientists observe that the moon is not made of green cheese, we should accept that truth. If experiments show that heavy objects fall the same speed as light objects, we should accept that truth. It is the latter category that I think we should call Science - the things that have a special authority, that force us to accept them as being true no matter how much we may not want it to be true.

I think this category of evidence is limited only to (1) that which we can observe in objective, repeatable, measureable experiments, and (2) anything which directly follows from those observations. And the reason this has a special authority is because it is really just observation. If we doubt it, we can try it for ourselves. And if we still doubt it afterwards, we can try it again and again. If the results are always the same, you must accept it, or deny your senses. This is the sort of "Science" that I think is rock solid. You cannot dispute it, as long as the experiments continue to support it. And if your religion disagrees with it, you should probably reject your religion - because that would mean you religion conflicts with what you can repeatably observe to be true.

I'd contrast this to a whole bunch of other sorts of conclusions that may be related to or based in part on scientific results, but are also based on other assumptions. These are extrapolations or extensions, where people take their own biases and opinions, and look at scientific results through that lens.

There's a lot here to reply to.

I don't think you have made (or can make) any useful distinction as a rule between these two 'categories' you describe. What I mean is that for any given published conclusion in science there is no reliable earmark to tell you whether or not it is this special-authority type of conclusion.

Your attempt to draw this distinction stems from a deep flaw in your entire understanding of what science is and what it does.

The type of science you relegate to "assumptions," "extrapolations or extensions" is a deeply important and extremely powerful component of science. It's how we knew the moon wasn't made of green cheese before Apollo 11. It's how we know evolution is a fact, and how we know so much about how it worked 500 million years ago. It's how we know anything about anything when the exact specific attribute isn't necessarily physically present and measurable (which is most of the time, if you're studying anything remotely interesting).

I've read enough of your posts to know that you find this aspect extremely difficult to grasp and even harder to accept. Science proceeds by firmly established logical footsteps from knowns to conjectures -- but when the logical footsteps are clear and solid enough, the conjectures are really as good as knowns.

By 'as good as,' of course I mean that (a) practical predictions can be made against them, and (b) further conjectures can be built on top of them.

Are flaws ever found in this superstructure of conjecture? Of course. Nature constantly surprises. (For this reason, among others, I am shy about making claims of truth, certainly in the context of Mathematician's initial inquiry here.)

Indulge me an illustration. When I was a kid, my grandfather built for us a toy, that resembled a box. A solid wooden box, about 28 inches long, 15 inches wide, and 5 inches tall. It was featureless except for two round holes -- one on each long side -- and two or three levers and knobs positioned apparently at random. The box came with a ball, a plain red rubber ball a little bigger than a ping-pong ball.

My grandfather dropped the ball into one hole.

Three weeks later my sister and I got the ball to fall out the other hole. Of course we immediately put it in the first hole again, and so on. After a few transits of the box, I could describe the innards of the box exactly: here a dead end, here a ramp, here a door, here a counterweight, here a revolving door, etc. I think I could almost diagram the inside of the box today, even though I never saw it or opened it.

Into which "category" did these conclusions fall: "objective, repeatable, measureable experiments, and ... anything which directly follows from those observations," or "assumptions," "extrapolations or extensions"? Clearly both (I think you would agree). In actuality, no such "categorization" exists in science.

Edit: clarity, grammar
 
Posted by John Van Pelt (Member # 5767) on :
 
quote:
I wrote:
no such "categorization" exists in science

I hasten to add that of course there are valid distinctions and categorizations between real or trusted science and junk science, non-science, philosophy, superstition, religion, wishful thinking, politics, advertising, and fraud.

It's just that these distinctions are not the ones that Tres thinks they are.

They are, however, extremely important to observe and identify, and often not easy to do so. I think the current climate change debate is an excellent example of the tremendous complications inherent to drawing such distinctions. And yet, in the case of climate change, many are using science to push social and political agendas. If their science is right, they are probably right to do so. But it puts a tremendous burden on officials and voters to sort out some very difficult questions.

Tres points at one valid distinction, which is buried in between "assumptions" and "extrapolations or extensions" -- to wit: "biases and opinions." Scientific journals and scientists work hard to detect and eliminate pure bias.

I'm not aware of any panel that has claimed to arrive at a scientific conclusion about when a fetus becomes a person. I guess I wouldn't be surprised if there were something of the sort somewhere. But I imagine most responsible scientists would examine such a claim very carefully. Did it, first of all, create a defensible definition for 'personhood'? Such a definition could not be scientific, but only philosophical (or religious, or semantic, you choose). But once established as a premise, certainly an answer of sorts could be arrived at.

Then you could say: "if you agree with the premise, then these scientific facts support the conclusion that a fetus becomes a person at moment X."

(The facts might be such things as measurements of brain waves, evidence of dreaming or desire or pain or consciousness, etc.)

Ironically, the "evidence" in such an argument could very well be precisely and only the sorts of "objective, repeatable, measureable experiments, and ... anything which directly follows from those observations" that Tres prefers as "authoritative" science. And yet the conclusion could only be considered scientific in the very narrowest sense -- that is, conditional upon acceptance of a totally non-scientific premise (definition of a person).

Of course, science reporting being what it is, a headline could well appear claiming "Scientists Agree Life Begins at Birth" (or Conception). But a full critique of science journalism is another topic. Suffice it to say that the firestorm of debate that would follow such a headline would be almost entirely misguided.
 
Posted by Orincoro (Member # 8854) on :
 
quote:
Originally posted by Dagonee:

How on earth do you think citing philosophers spending terrific energy trying to define terms somehow refutes what Tres said, when he himself commented on the how much energy they spend trying to define terms.

Because the disagreement is the source of so much insite in philosophy, and especially in my own field of interest lately- literary criticism. A philosopher or a critic can spend an entire treatise attempting to identify the meaning of a single word, in a single context. He does not expect others to agree, nor probably does he hope to absolutely prove some point, but merely to build a universal understanding around one concept, in one usage. That universal understanding, though most believe you can't achieve it, is the goal for some, and the end of the world for others.

I wished, and do wish, to point out that philosophers don't START at the assumption of an agreed upon understanding, they simply use that as their essentially unatainable goal. The universal understanding is transcendental, it is never complete, and it never starts complete, even if you think you are writing from a common understanding.

I suppose, now that you mention it, Tres saying that common understandings are essential to philosophy is not the same thing as saying that these understandings, these common definitions are actually PRESENT in a dialogue, however that is the impression I got, and that is the direction his post seemed to lead. If that's wrong, he or I or both of us should do a better job of being clear about this distinction, because its the basis of the disagreement. However, if he does agree with me, then he has contradicted himself by saying that common definitions are important goals and useful tools for thought and dialogue in any discipline. His interest in debunking astronomy based on the haggling over jargon tells me that he doesn't appreciate this important aspect of dialogue. Could be wrong, but I don't think so.

Yeah, I'm rude, I don't feel a need to apologize- especially when its your pot calling my kettle, so please stop trying to referee my side of any discussion. [Wink]

edit: What tires me in dealing with Tres in most discussions is that your attempts to appeal to different aspects of the issue, to get him to start applying some of his own ideas and seeing if they work, bounce off like he's made of some kind of very powerful rubber compound. Anything you ask for, any consequence of his position you point out is met with "I just don't believe that," which is frustrating to someone who would be willing to consider the point and see where it leads. He may actually be attempting to do this, but each of his responses comes out as a restatement of the original idea, starting with a flat dismissal of a point he ought to consider a little more carefully. "That's not true," is a common leader, even when the point is highly debatable and often subjective.

Given these respones, which are not entirely rude on the face of them, but which do no justice to the discussion, I get frustrated, and my rudeness looks like rudeness. Often though, especially now, I feel I am returning the feeling in kind, even if Tres doesn't come out and say: "Your being a complete jackass," which I am perfectly willing to do.

Edit yet again: Though I hate to play drive-by poster troll, I am moving to my new apartment this morning and won't be on for the weekend, so if I don't answer your plaintive calls for my appearance at trial, forgive me once again. [Wave]

[ August 31, 2006, 06:01 AM: Message edited by: Orincoro ]
 
Posted by rivka (Member # 4859) on :
 
quote:
Originally posted by John Van Pelt:
quote:
Rivka wrote:
But because [ID] claims "God did it, and science can prove that He did"

This isn't quite accurate. ID claims, "(a) Something purposeful (OK, call it God) did it, (b) science hasn't proven that it/he didn't, (c) many of the gaps/unknowns/conundrums in current evolutionary knowledge cannot (ever) be solved within science, and thus explanations invoking a designer must be invoked, and (d) the claim presented in (c) requires no scientific proof because it is patently self-obvious."
Ok, so I oversimplified. [Wink]
 
Posted by Avin (Member # 7751) on :
 
This is a great thread, Mathematician, and unfortunately there is too much going on for me to pick something to respond to.

But I agree with your line of reasoning completely. And to answer one of your questions above, I don't value reason above faith, in all cases. Of course it's somewhat hard to assert my own consistency in this (after all I cannot prove myself to be consistent!) but I currently hold the beliefs I do, that of a Young-Earth Creationist, precisely because I value certain propositions to a higher degree than I value the scientific theories that have been made to explain the evidence we see. In my mind, as long as I can see that under my system there may be a reasonable explanation for the same evidence, then I am satisfied. I didn't always think this way - I shared the prevalent view in this thread that logic was indeed superior or that the Bible spoke to different things than faith, but actually started into the same line of questioning while pursuing my Math degree. I came to the conclusion that to take additional items on faith that the scientific world does not was perfectly acceptable to me as long as I did not get inconsistent results, much like I was adding the Continuum Hypothesis to ZFC or some such thing.
 
Posted by Mathematician (Member # 9586) on :
 
quote:
Originally posted by Avin:
Of course it's somewhat hard to assert my own consistency in this (after all I cannot prove myself to be consistent!) but I currently hold the beliefs I do ...

Not to be picky, but you may have hit on a pet peeve of mine. If you were NOT referencing Godel's proofs in stating that you can't prove yourself to be conistent, ignore the rest.

If so....

When applying mathematical theorems like this to life, you have to be REALLY careful. Godel's incompleteness theorems (like all theorems), have that pesky hypothesis part. Mainly, Godel proved that his incompleteness theorems hold if

1) The system can be modeled as formal logic

2) The system has an "effective algorithm" for determining whether or not a random string of characters really is allowable sentence, formula, etc, according to the rules of the formal logic.

3) The system is strong enough to develope multiplication.


The big thing is that NO ONE is certain whether thought processes as a whole CAN be modeled in the language of formal logic. If not, it's entirely possible that consistency CAN be proved.

I think 2) is satisfied in most peoples minds (you can tell whether or not your thoughts are coherent), but this is of course debatable.

Finally, 3) may not be satisfied. Sure, you can multiply, but that doesn't mean you can DEVELOPE multiplication in the formal language.

(Interesting aside - Presburger Arithmetic satisfies 1) and 2), but for 3), instead only allows for addition/subtractoin. Presburger proved his own arithmetic was both complete and consistent)
 
Posted by Avin (Member # 7751) on :
 
I'm aware of those restrictions to the theorem, since I have rigorously seen his proof applied in a Type theory system, doing parts of it as exercises myself. I wasn't making the assertion that my internal reference was equivalent to an axiomatic system (conditions 2 and 3 are a bit more straightforward) but I was merely referencing the seemingly obvious philosophical notion that a person cannot know that his thoughts are consistent: they can only discover an inconsistency and perhaps revise their thoughts as a result. So the specific formal logical proofs used to demonstrate the incompleteness of a system does not apply directly but it has the same intuitive result.
 
Posted by Tresopax (Member # 1063) on :
 
quote:
edit: What tires me in dealing with Tres in most discussions is that your attempts to appeal to different aspects of the issue, to get him to start applying some of his own ideas and seeing if they work, bounce off like he's made of some kind of very powerful rubber compound. Anything you ask for, any consequence of his position you point out is met with "I just don't believe that," which is frustrating to someone who would be willing to consider the point and see where it leads. He may actually be attempting to do this, but each of his responses comes out as a restatement of the original idea, starting with a flat dismissal of a point he ought to consider a little more carefully. "That's not true," is a common leader, even when the point is highly debatable and often subjective.
In defense of myself, there are reasons why I argue in the way I do. I often take positions that walk a fine line between two established viewpoints, or approach a familiar position in an unfamiliar way, by challenging certain assumptions that may be very widely accepted but that I don't think are well justified. Several things result from this:

Firstly, people seem to constantly equate the very specific position I take with other more common positions that sound sort of like it. For instance, in this thread it has been suggested that I agree with ID when I don't, that I intend to "debunk astronomy" when I am doing the reverse, that I think definitions are not useful to science when I think they are essential to science, etc. As a result, I feel the need to restate my position to attempt to make it clear exactly what I am and am not arguing.

Secondly, people often try to refute my position starting from assumptions that I don't agree with. In such cases, I have to say "I just don't believe that" because I don't. Please notice that I always at least try to give some explanation/reason why I don't believe it. But if I don't believe that shared definitions are unimportant to philosophers then I'm not going to go down a length path of discussion that is entirely built upon that assumption. If you want to convince me of your viewpoint, you need to start with assumptions I do believe in, or show me why I should accept the assumptions that I reject.

Thirdly, often people make points that I DO agree with, but then I realize that I can improve my position in some way by taking their argument into account. This is another reason I may have to restate my position - in order to revise it to fit some new information, rather than reject it.

I don't flatly dismiss your arguments though. I may state that I flat out disagree with it, but that does not mean I didn't consider it or that I have no reason for disagreeing. I don't think any of this is rude. Even if you do find it rude, though, I still don't think that justifies making invalid ad hominem arguments against my position. For one thing, I have had several philosophy professors whose judgement in grading would be called into question if I truly know as little about these issues as you have tried to suggest I do - certainly my Philosophy of Science professor!

quote:
However, if he does agree with me, then he has contradicted himself by saying that common definitions are important goals and useful tools for thought and dialogue in any discipline.
I don't think this contradicts me - I don't think the things I've said imply that definitions are not important or not useful. They are important! I just don't think the discussion of which definitions are best should be considered doing science, even when the terms are scientific.

quote:
Three weeks later my sister and I got the ball to fall out the other hole. Of course we immediately put it in the first hole again, and so on. After a few transits of the box, I could describe the innards of the box exactly: here a dead end, here a ramp, here a door, here a counterweight, here a revolving door, etc. I think I could almost diagram the inside of the box today, even though I never saw it or opened it.

Into which "category" did these conclusions fall: "objective, repeatable, measureable experiments, and ... anything which directly follows from those observations," or "assumptions," "extrapolations or extensions"? Clearly both (I think you would agree). In actuality, no such "categorization" exists in science.

I would say the experimenting you did and the "theory" you generated about what the inside looked like was all experimentation and conclusions that directly followed from that experimentation. And as long as you are just testing a theory, and don't claim your model of the inside of that box is a "fact", I don't think you are making any unscientific leaps. This is because it the model you came up with is consistent with (not falsified by) all the little tests you did on the box. The extrapolations or assumptions are more like "Why does this box have the shape it does?" or "Will my theory always hold true?" or "Is it good to play with the box?" etc - they might rely on parts of my theory, but the logic for them depends also on other assumptions that don't stem from our experimentation.
 
Posted by John Van Pelt (Member # 5767) on :
 
quote:
Tres:
I just don't think the discussion of which definitions are best should be considered doing science, even when the terms are scientific.

Do you see how this admission undermines the point you originally tried to explore in the other thread?

We all agree that discussions of definitions are not part of doing science as such -- that is, not uniquely or even distinctly part of the defining nature of science (as opposed to religion, philosophy, shoemaking, etc.).

ALL endeavors include discussions of definitions. The fact that scientists engaged in a protracted 'discussion of definitions' therefore does not at all belie scientists' earlier claims used to exclude ID from public-school classrooms.
quote:
The extrapolations or assumptions are more like "Why does this box have the shape it does?" or "Will my theory always hold true?" or "Is it good to play with the box?" etc.
I thought you might misinterpret this point. Let me be more explicit.

The "experiment and conclusions" I and my sisters did were FULL of extrapolations and assumptions, every step of the way. There is no possible way we could have solved the puzzle without them.

A simple part of it might go like this:
code:
"I can hear the ball rolling. It appears to roll
about halfway, and encounter an obstacle." (Observation)

"Maybe the maze turns to the left at that point." (Extrapolation)

"Let's tilt the box to the left and see if we hear
it roll some more." (Assumption [that we will be
able to detect additional progress by the ball, if
it occurs])

Tilt box. (Experiment)

There must have been 50 or more operations of that sort in arriving at the solution, and none of them make any sense whatever without extrapolation and assumption.
quote:
as long as you are just testing a theory, and don't claim your model of the inside of that box is a "fact...
This is precisely what we are talking about here. I do claim -- within certain limits that I can define very precisely -- that my model IS fact. Just as I can claim, in the above example, if I do hear the ball roll to the left, that my extrapolation and assumption about the direction of the maze was correct. The entire solution is built out of 50 individual 'proven' (or disproven) conjectures, and the whole is confirmed when the ball exits the box.

My mental model for the inside of the box IS fact, provided you understand that it is fact within the domain of the puzzle-box itself. This is a crucial distinction. Does the puzzle box work the same with a marble? Does it work the same underwater? If I built one myself, would all my materials and dimensions exactly match my grandfather's? Those are different domains, and my 'knowledge' is not comprehensive enough or fine-grained enough to illuminate them (new experiments would be needed to expand into those areas, which would be perfectly doable). But within the scope of that ball and that puzzle, I am completely sure my model is accurate.

What is the domain within which evolution is known to be a fact? The domain of biological nature, as revealed in life forms, genetics, biochemistry, paleontology, geology, etc. Within the aggregate of those domains, evolution is acknowledged to be absolutely fact, with a pretty comprehensive and fine-grained understanding of its mechanisms (although this is constantly under development and refinement).

Edit: missing word, typo, markup

[ September 01, 2006, 01:20 PM: Message edited by: John Van Pelt ]
 
Posted by John Van Pelt (Member # 5767) on :
 
I might add, in the spirit of this thread, that a possible definition for an endeavor that seeks 'truth' (or 'Truth') is one that would apply outside or beyond any definition of a domain (as I am using the term above).

Religion, at a certain level, makes claims about the ultimate origin and nature of all things, both physical and metaphysical. It deliberately overreaches any domain constraint. Does a fundamental Christian believe that a Muslim is damned or saved according to the same rules as Christians (and vice-versa)? Yes.

Some lower-level areas of philosophy and religion do allow constraint to a given domain -- for example, a discourse on morals and ethics probably pertains to humans, and not animals and plants, much less inanimate matter. This opens the door very naturally to relativism of various sorts -- can a baby lie? should a mentally retarded person be executed for murder? Can someone forgive another for a perceived wrong, acknowledging that their value systems differ? and many more.

To the extent a system of thought can (in the case of philosophy) or must (in the case of science) be bound to a domain in order to remain self-consistent and 'true', is the extent to which that system of thought escapes any claim to 'Truth.'

Some people have the impression that science does operate at that level: above all domain constraint. Some people think science says 'there is no god,' or 'everything in the universe is matter' -- and accepting that science makes these claims (which it doesn't), people object that science arrogates to itself a definition of 'Truth.'

Science produces facts, not truth, which are only useful or even meaningful in particular contexts, contexts that necessarily exclude significant areas of discourse (such as faith).
 
Posted by dkw (Member # 3264) on :
 
quote:
Some people think science says 'there is no god,' or 'everything in the universe is matter' -- and accepting that science makes these claims (which it doesn't), people object that science arrogates to itself a definition of 'Truth.'
Some people (and some scientists even) believe/state that "science" does make those claims. I agreee with you that it doesn't, but that doesn't change the fact that the people who are making the objections are not making it up out of nothing.

Better education about what science is and isn't is needed all 'round, I would say.
 
Posted by John Van Pelt (Member # 5767) on :
 
I agree on all points, dkw.
 
Posted by Tresopax (Member # 1063) on :
 
quote:
We all agree that discussions of definitions are not part of doing science as such -- that is, not uniquely or even distinctly part of the defining nature of science (as opposed to religion, philosophy, shoemaking, etc.).

ALL endeavors include discussions of definitions. The fact that scientists engaged in a protracted 'discussion of definitions' therefore does not at all belie scientists' earlier claims used to exclude ID from public-school classrooms.

I don't see how this undermines my original point in the other thread, namely that science (and scientists) are concerned with more than just doing science, in the strict sense.

I think the fact that all endeavors include discussions of definitions only proves only that all endeavors, including science, are concerned not just with the endeavor itself but also things related to that endeavor.

quote:
"I can hear the ball rolling. It appears to rollabout halfway, and encounter an obstacle." (Observation)"Maybe the maze turns to the left at that point." (Extrapolation)"Let's tilt the box to the left and see if we hear it roll some more." (Assumption [that we will be able to detect additional progress by the ball, if it occurs])Tilt box. (Experiment)
I would not consider those assumptions or extrapolations, though, because you are not assuming them to be true without testing them. Instead, they are a hypothesis, which is a part of the scientific method. If you test it, and keep it as your working theory as long as no test falsifies it, then I think that is all doing science.

I think an assumption would be if you heard the ball rolling and thus concluded that it must turn left at that point, and assumed that to be true without testing it.

quote:
This is precisely what we are talking about here. I do claim -- within certain limits that I can define very precisely -- that my model IS fact.
Then that is a point where I think you have left the boundries of what science can actually say.

quote:
What is the domain within which evolution is known to be a fact? The domain of biological nature, as revealed in life forms, genetics, biochemistry, paleontology, geology, etc. Within the aggregate of those domains, evolution is acknowledged to be absolutely fact, with a pretty comprehensive and fine-grained understanding of its mechanisms (although this is constantly under development and refinement).
I don't think evolution, as a whole, is known to be a fact, within those domains or anywhere. It is just the preferred model that currently fits all (or at least the vast majority of) the known data. It could one day be falsified, and thus turn out to be false. Couldn't it?

quote:
Science produces facts, not truth, which are only useful or even meaningful in particular contexts, contexts that necessarily exclude significant areas of discourse (such as faith).
Why do you say science doesn't produce truth? I would say science is only useful insofar as we believe scientific theories are true.

I'm also not sure what you mean by a fact that is not truth. Don't facts have to be true?
 
Posted by orlox (Member # 2392) on :
 
Evolution can be observed.

Natural selection is a theory to explain how and why evolution occurs. It is up for debate.

Facts are not the universe, they are our understandings of the universe and thus dimensionally flawed.

The sun rises every morning.

But of course that is not true.
 
Posted by Dan_raven (Member # 3383) on :
 
Jumping back to Avin:

You are free to be a Young Earth Creationist based on nothing but faith. The question remains, what do you do when you meet an Old Earth Hindu who believes the world has come and gone 10,000,000 times? He also has his faith. How do you satisfy these questions of conflicting beliefs?

One way, the historic way, is to force those who disbelieve into beleiving in what you have faith is the correct way. This created such a wave of death and destruction in the renaisance that a more logical solution was envisioned--science.

You say x. I say y. We can either both hold firmly wto what we say, or we can decide to use logic to determine which is correct.

Did you ever notice, you can teach Evolution without mentioning Religion or ID. You could not teach ID without mentioning Religion or Evolution.

Well, you could. The class would be:
"Something made everything the way it is now."

Which would be followed by questions:

"How?" "We don't know and can't know, ask the leader of your faith."
"Why?" "We don't know and can't know, ask the leader of your faith."
"Who?" "We don't know and can't know, ask the leader of your faith."

Not to difficult a class to ace.
 
Posted by Avin (Member # 7751) on :
 
quote:
How do you satisfy these questions of conflicting beliefs?
From a strict epistemological view, you really cannot. If my belief system happens to preclude being forced or re-evaluation due to any form of reason, then really I can only change it by my own choice. And although I am not above reconsidering beliefs I hold based on rational argument from my own assumptions and by showing I am inconsistent about something, you will not be able to convince me that my beliefs are wrong using rational argument based on any external assumptions, such as your own.

So the situation is you say y, I say x. You think that because we disagree we need to use logic to determine which is correct: fine by me, but if your logic starts from assumptions contrary to what I already hold, then of course I am not going to accept your logic.

And really, if I refused to listen to your logic at all, who are you to say that I am wrong? Logic cannot dictate right and wrong. I have stated that I try to be self-consistent; but if I decided to deliberately believe in inconsistent thoughts, by what standard could you argue against me?

I find it somewhat ironic that although Christianity is an evangelistic religion, you seem to be the one evangelizing here. I do believe in defending and sharing my faith when appropriate using reason to justify my beliefs, but I do not believe it possible for that reason to convince everyone.

[edit] I see you edited your post after I responded. Please note that I am NOT a supporter of Intelligent Design and do not think it a very good argument at all. On the other hand, you could very easily teach Creationism without mentioning evolution (although I certainly believe in evolution and intend on teaching my kids all about it as well) and you could also easily answer all those questions.
 
Posted by Tresopax (Member # 1063) on :
 
quote:
And really, if I refused to listen to your logic at all, who are you to say that I am wrong? Logic cannot dictate right and wrong. I have stated that I try to be self-consistent; but if I decided to deliberately believe in inconsistent thoughts, by what standard could you argue against me?

Your own. If you are believing in inconsistent things, then that means you are believing things that you believe are wrong.

quote:
Did you ever notice, you can teach Evolution without mentioning Religion or ID.
The trouble with that is that I can teach Christianity without mentioning Hinduism, but by teaching Christianity I am still nevertheless probably calling Hinduism wrong, even if I am not specifically saying it.
 
Posted by Dan_raven (Member # 3383) on :
 
Avin, sorry about the edit. I didn't realized you had responded. I had hit the reply key too soon.

I don't doubt that you and most Christians have no problem living with people who do not share your beliefs. You won't try to force them on others. Unfortunately history shows that the minority of believers that do try to force thier ideas do so with bloody results.

I do not limit those believers to Christians. Whether its Muslims, Hindus, or Aztecs, some people in most religions will be violent in spreading their beliefs. Science took form as an answer to that self destructive tendancy.

My jump to ID was not meant to be directed to you, but to the whole ID debate.

I know that you could teach Creationism without mentioning Evolution. Then again, Creationism doesn't claim to be science. It is religion.

Tresopax--Yes, but teaching science you are saying that some beliefs are wrong. But science classes are not about beliefs, they are about science. To debate the philosophy behind science, and the particular religions who are apparently attacked by it, then take ID to philosophy class.
 
Posted by Tarrsk (Member # 332) on :
 
Avin- A personal question, if that's all right. You identify yourself as a Young Earth Creationist, but also "believe in evolution." We've established in this thread that it is certainly possible to believe that God directed evolution, and that theistic evolution does not any in any way contradict current science. However, I'm a little confused how one would reconcile Young-Earth creationism with evolution... if the Earth has only existed for a few thousand years, there really hasn't been enough time for evolution to do it's thing. Please correct me if I'm confused about your beliefs; I'm honestly curious here. [Smile]
 
Posted by John Van Pelt (Member # 5767) on :
 
quote:
Tres:
I don't see how this undermines my original point in the other thread, namely that science (and scientists) are concerned with more than just doing science, in the strict sense.

(Um, yes. And also concerned with eating, and going to the bathroom, and wondering whether it will rain on their picnic.)

Your original point was that:
(1) during the ID debates, scientists claimed that the business of science and scientists was only that which could be proven via the scientific method (I am paraphrasing),
(2) a debate (undertaken by scientists) about the definition of the word 'planet' did not fall into that category,
(3) therefore the business of science and scientists must include a larger range of activities, including not just that which can be proven via the scientific method, but also the defining of terms, and perhaps other things,
(4) and therefore those scientists cited in (1) used an erroneous argument to disqualify discussions of ID from science.

Several of us replied with the news that can't have seemed anything but blindingly obvious to everyone but you, namely that the defining of terms (along with deciding what to eat for lunch and complaining about the weather) is something that everybody does all the time, is not a part of science per se, and whether a scientist engages in the defining of terms has no bearing whatever on how we may interpret what is the rightful domain of science and scientists.

Several posts later you say "I just don't think the discussion of which definitions are best should be considered doing science, even when the terms are scientific," which directly contradicts step (3) in my version of your argument above.

Do you put the defining of terms among those activities that distinctly characterize scientific activity? Or do you not? You can't have it both ways.
 
Posted by Mathematician (Member # 9586) on :
 
quote:
Originally posted by Tarrsk:
Avin- A personal question, if that's all right. You identify yourself as a Young Earth Creationist, but also "believe in evolution." We've established in this thread that it is certainly possible to believe that God directed evolution, and that theistic evolution does not any in any way contradict current science. However, I'm a little confused how one would reconcile Young-Earth creationism with evolution... if the Earth has only existed for a few thousand years, there really hasn't been enough time for evolution to do it's thing. Please correct me if I'm confused about your beliefs; I'm honestly curious here. :)

I don't mean to answer to Avin, but I can offer my own view point on it.

I have read books/articls by Gerald Schroeder and by some other guy (I don't remember his name) which address this. For both of them, the idea can be summarized as follows:

When people ask how old the universe is, there's a piece that's missing - Einstein taught us that time is relative to your location (among other things). Thus, it is much more accurate to say "how old is THIS piece of the universe".

The idea I've read is as follows: what if according to SOME clock, 6 days tick by while according to another 15 billion years pass by? Schroeder addresses this in "The Science of God" among other books. He even works out some of the math (poorly), and gets relatively decent results. I recently read a paper by him that I found very interesting (sorry, I know NO html or stuff like that):

http://aish.com/societywork/sciencenature/Age_of_the_Universe.asp

The other guy (I REALLY wish I could remember his name), wrote a scientific paper addressing it. He challenged one (currently accepted) view point of cosmology - that there is no center of the universe. He said, what if there IS a center, and the solar system is roughly there? Further, what if at the moment of the big bang, when all the matter is super concentrated, there was a white hole (similar analogue to a black hole, except it EJECTS everything rather than trapping it. And they are on as firm a ground as black holes, at least theoretically). Then what happens?

1. The white hole pushes everything out (universal expansion)

2. As the white hole shrinks, eventually the Earth gets near the event horizon. White holes work just like black holes - at the event horizon, time virtually stops. Thus, it is EASY for incredible amounts of time to pass for the rest of the universe and at the same time, relatively none pass for earth.

The guy who wrote this paper believes in Theistic evolution. The part where God comes in is this: The mere probability of the Earth being at the center (if there even is a center) is incredibly small. God just kind of started things out there.

The paper, while rather technical, isn't what I'd hoped. He starts with the Schwartzchild equation from general relativity (the one that makes people think there are black/white holes) and goes from there. The problem is, in a single paper, it's impossible to address everything of importance (such as molecule production/distributions, cosmic microwave background radiation, etc).
 
Posted by John Van Pelt (Member # 5767) on :
 
quote:
quote:
quote:
"I can hear the ball rolling. It appears to rollabout halfway, and encounter an obstacle." (Observation)"Maybe the maze turns to the left at that point." (Extrapolation)"Let's tilt the box to the left and see if we hear it roll some more." (Assumption [that we will be able to detect additional progress by the ball, if it occurs])Tilt box. (Experiment)

I would not consider those assumptions or extrapolations, though, because you are not assuming them to be true without testing them. Instead, they are a hypothesis, which is a part of the scientific method. If you test it, and keep it as your working theory as long as no test falsifies it, then I think that is all doing science.
Then you are wrong. Hypotheses and 'working theories' are assumptions -- and the expression I listed as an extrapolation is most definitely an extrapolation.

I have no argument about the testing and falsification stuff. My primary point was to try to help you see that assumption and extrapolation (yes, of course as waystations on the way to further testing) are integral to science, not hallmarks of non-science as you claimed.
quote:
I think an assumption would be if you heard the ball rolling and thus concluded that it must turn left at that point, and assumed that to be true without testing it.
I guess you would disagree that a working assumption is an assumption?*** Again, I was simply pointing out that there is a vital role for assumptions in science, which you originally denied.

If for you 'assumption' strictly means 'assumption that remains forever untested' or 'assumption that is announced as fact without having been tested,' then you are right, those kinds of assumptions have no place in science, and few would disagree with that.

***Let's look at another example with the puzzle box. Let's say my grandfather had built the ball maze inside the box to have two levels: an upper course and lower course, overlaid one on the other, perhaps connected by a slanted section, like a parking garage.

But let's say I had not yet figured this out, so when the ball seems to be rolling the length of the box I imagine it is in one channel, even though sometimes it is rolling down an upper channel, and at other times rolling down a lower channel.

With me so far?

There are actually two forms of assumption at play here. One is a largely unconscious assumption that the maze is on one level. The other is a more particular, but related, assumption that when the ball is rolling here, it is always in the same part of the maze.

Either assumption can be dashed without ever explicitly acknowledging the assumption as such and testing for it. But you could also have an AHA! moment and say, "wait a sec, wiley ol' grandpop, is this on more than one level?" and very likely a whole bunch of divergent observations that weren't making sense click into place. Again, you haven't tested for the assumption specifically, or rather you did, earlier, unintentionally.

So, I think I have described a situation, that I consider a very good example of scientific method, where "assumption would be if you ... concluded [something] ..., and assumed that to be true without testing it" definitely takes place, and where the assumption could persist, without testing, for a considerable period of time. And yet this is all essential to science, contrary to what you seem to believe.

[ September 01, 2006, 08:09 PM: Message edited by: John Van Pelt ]
 
Posted by John Van Pelt (Member # 5767) on :
 
A side point, but important to an understanding of scientific method and of how 'factual' modern evolutionary theory is.
quote:
Tres wrote:
If you test it, and keep it as your working theory as long as no test falsifies it....

This phrasing suggests a common misconception, that the falsification of a hypothesis is somehow always possible in the distant future, even if it tests true at present.

Now, on the one hand, of course I recognize that history is littered with facts that were once deemed true but which have since been disproved. So it would smack of hubris (or stupidity) to claim that kind of infallibility for science.

And in fact, as I have noted a number of times, real science thrives on this. The advancement of knowledge often -- maybe even usually -- requires a certain appetite for changing one's mind or being proved wrong.

On the other hand, and this is very important, there are lots of logical/deductive constructs that allow an experimenter to formulate closed-end hypotheses; that is, instead of:
quote:
I believe X, and so far X seems to be true, but if X is falsified, I don't yet have any alternative in mind...
...this:
quote:
The answer can only be X, Y or Z. Y and Z have been falsified, therefore it must be X.
There are other logical/deductive forms that can produce similarly positive results.
 
Posted by Mathematician (Member # 9586) on :
 
quote:
Originally posted by John Van Pelt:

On the other hand, and this is very important, there are lots of logical/deductive constructs that allow an experimenter to formulate closed-end hypotheses; that is, instead of:
quote:
I believe X, and so far X seems to be true, but if X is falsified, I don't yet have any alternative in mind...
...this:
quote:
The answer can only be X, Y or Z. Y and Z have been falsified, therefore it must be X.
There are other logical/deductive forms that can produce similarly positive results.

In terms of observational science, I agree. But when it comes to experimental science, I disagree with this. I agree that given certain beginning assumptions, sometimes one can arrive at the conclusion "X is the only possibility", but I think the assumptions can ALWAYS be questioned.

That's not to say that some assumptions aren't extraordinarily reasonable. But sometimes extraordinarily reasonable assumptions turn out to be wrong (e.g. who would have thought that time wasn't an absolute?).
 
Posted by John Van Pelt (Member # 5767) on :
 
quote:
Tres wrote:
I don't think evolution, as a whole, is known to be a fact, within those domains or anywhere. It is just the preferred model that currently fits all (or at least the vast majority of) the known data. It could one day be falsified, and thus turn out to be false. Couldn't it?

I think you should read up on evolutionary theory, starting with Darwin and ending with Gould.

It is most certainly not "just a preferred model," and as much as I, as an open-minded person, resist the word "never," I do believe the general outline of evolutionary theory and most of its particulars will not be falsified, with much the same certainty as I might say "the moon is not made of green cheese" will not be falsified.

For the record, I do not consider this an article of faith on my part; I have read widely on the topic in both lay and professional publications, and I remain deeply convinced by the sheer power of its explanations.
quote:
Why do you say science doesn't produce truth?
I think I explained more about this point of view in a later post than the one you quoted. I was probably using somewhat sloppy language, but the main idea is that I feel the word 'truth' conveys an absolutism that is rarely applicable in science.

Consider the argument about how common extraterrestrial life may be in the universe. Early estimates looked at the mind-boggling vastness of space, the millions of galaxies with their billions of stars, and asked, "How could it be that Earth is the only planet to have spawned life?" A more recent (and controversial) study looked at the specific conditions necessary to the stability of an earth-like planet -- not too close to the galactic center, not too far, not too near violent astral events, etc. That study suggested that earth-like planets, even in the vastness of all space, could be quite rare.

The thing is, nothing in all that has much to do with 'truth.' Lots of facts, sure, but not truth.
quote:
I would say science is only useful insofar as we believe scientific theories are true.
So if I believe a theory that says gravity is powered by chamomile tea, it becomes a useful theory? :-)

Science is useful in a number of different ways.
(1) Science as engineering has practical utility: we can build bridges, and they (usually) don't fall down.
(2) Life sciences including evolution, genetics, not to mention medicine, give us information about how the natural world works, and how we (as uniquely intelligent and uniquely destructive creatures) impact it and each other. This guides our behavior, if we let it, for good or ill.
(3) Scientific theories that are 'agreed' to be factual -- where there is strong consensus -- where you might say "we believe it's true" -- these have a special utility: they can be treated as axiomatic in the development of additional theories.

In all cases, it is not so much to do with what we 'believe,' but what the consensus is. Conclusions that have strong consensus, that have been proved many times over, that are supported by unpredicted and surprising results in other arenas -- these come close to being treated as true.

I think some of the writings of religious scientists would be interesting here -- to see whether they actually disbelieve their science (where it conflicts with religion), but nevertheless from a practical standpoint treat the facts as true.
quote:
I'm also not sure what you mean by a fact that is not truth. Don't facts have to be true?
(I'm glad you're asking some of this, it is making me think.)

I think I was getting at a crucial aspect of scientific culture, and it's this: a scientific 'fact' comes with a certain amount of baggage. Somewhere behind every scientific fact is a provenance of idea, experiment, proof, and so forth. Part of science is that you are always allowed to ask "sez who?" or "how come?"

This has two important consequences:

1. Every fact comes with the tacit message, "figure it out for yourself, see if you think it's true, too."

2. Every fact comes with the tacit message, "some one of the assertions or discoveries that undergird this fact could be mistaken." If such a mistake is found, it won't necessarily destroy the truth of this fact, but the possibility (in a very generic sense) exists.

Neither of these characteristics sound to me like a discipline that produces "truth," though I'll grant it is a subjective distinction.

I hope these ideas are useful in exploring this topic.
 
Posted by John Van Pelt (Member # 5767) on :
 
quote:
Mathematician wrote:
I agree that given certain beginning assumptions, sometimes one can arrive at the conclusion "X is the only possibility", but I think the assumptions can ALWAYS be questioned.

Yes, I foresaw this objection, and I agree. I was simply trying to open Tres's eyes to robust deductive structures used in science that are alternatives to the simple one that is so often cited by evolution skeptics ("it's only considered valid because it happens to fit the known data and hasn't [yet] been falsified").
 
Posted by Mathematician (Member # 9586) on :
 
quote:
Originally posted by John Van Pelt:
quote:
Mathematician wrote:
I agree that given certain beginning assumptions, sometimes one can arrive at the conclusion "X is the only possibility", but I think the assumptions can ALWAYS be questioned.

Yes, I foresaw this objection, and I agree. I was simply trying to open Tres's eyes to robust deductive structures used in science that are alternatives to the simple one that is so often cited by evolution skeptics ("it's only considered valid because it happens to fit the known data and hasn't [yet] been falsified").
Even along these lines, there is a still an issue. Even if we agree on our initial assumptions to begin with, science can merely say "the only SCIENTIFIC theory meeting these assumptions/data/etc can be X", it can not (by definition), rule out non-scientific "theories (answers/explanations)".

Thus, science may be able to say "really guys, the only reasonable scientific theory for life the universe and everything is evolution", but it MUST say "even though I've ruled out every scientific theory but evolution, evolution could still possibly be wrong (though we obviously believe that chance to be small enough to neglect)"
 
Posted by John Van Pelt (Member # 5767) on :
 
quote:
...it can not (by definition), rule out non-scientific "theories (answers/explanations)".

Thus, science may be able to say "really guys, the only reasonable scientific theory for life the universe and everything is evolution", but it MUST say "even though I've ruled out every scientific theory but evolution, evolution could still possibly be wrong (though we obviously believe that chance to be small enough to neglect)"

This is not where I thought you were going with this. (I thought we were talking about the inherent uncertainties of various deductive models.)

I take it as a given that science cannot rule out non-scientific theories, and I take it as a given that science accepts that fact. In fact, that is mainly what I meant earlier when I said "What is the domain within which evolution is known to be a fact? The domain of biological nature, as revealed in life forms, genetics, biochemistry, paleontology, geology, etc."

It IS important for scientists to think not only rigorously but also broadly when concocting hypotheses. Picture a botanist studying a newly discovered orchid and trying to determine its relation to other orchids and to its ecosystem. Would you say that she is obliged, in addition to various 'scientific' theories, to consider a theory like, "or, well, it might have just been designed and placed here by who knows who"? Especially when that exact explanation is the one that was already superseded, 150 years ago? Especially when, if one contemplates tackling that as a theory or explanation, one quickly realizes there is absolutely nothing in it to be tested or studied?

So yes, there is a very general obligation on the scientific community to humbly allow that we don't know every dam-q thing, and that the universe undoubtedly has surprises in store for us. But that doesn't change one whit the characterization of what is practically considered the consensus body of knowledge, how it is taught, or the direction that scientists take in doing original research.

In other words, the "issue" you cite is indeed "small enough to neglect" (by a wide margin), and is thus no issue.

Edit: typo
 
Posted by Avin (Member # 7751) on :
 
Sorry for the late replies, I was out of town for the weekend.

quote:
I do not limit those believers to Christians. Whether its Muslims, Hindus, or Aztecs, some people in most religions will be violent in spreading their beliefs. Science took form as an answer to that self destructive tendancy.
Personally I disagree that this is the reason why science was developed. I would agree that this is a contributing reason why scientific methology became so prevalent and modernistic thinking took root, but I sharply disagree that this was the purpose in mind for most science leading up to the Enlightenment.

quote:
I know that you could teach Creationism without mentioning Evolution. Then again, Creationism doesn't claim to be science. It is religion.
Agreed, although personally I don't much care too much for categories of knowledge.

Tarrsk -
quote:
Avin- A personal question, if that's all right. You identify yourself as a Young Earth Creationist, but also "believe in evolution." We've established in this thread that it is certainly possible to believe that God directed evolution, and that theistic evolution does not any in any way contradict current science. However, I'm a little confused how one would reconcile Young-Earth creationism with evolution... if the Earth has only existed for a few thousand years, there really hasn't been enough time for evolution to do it's thing. Please correct me if I'm confused about your beliefs; I'm honestly curious here.
When I say I believe in evolution, you'll perhaps have to qualify that. I believe in the process of evolution occuring throughout history (no distinction between macro and micro evolution, as some creationists like to maintain), I just have a shorter history to apply that in. Since I accept on faith first that God created various kinds of life on earth roughly 6,000 years ago, then evolution must have taken over from there. I take no issue with scientific findings that rely on evolution in the present, but the dating of various events, such as fossils, geological strata, or the light from distant stars I am forced to submit can be dated differently. Mathematician has mentioned some ideas on this already; while I am not confident in any one theory about this, I am suitably conviced that no one issue is completely unresolvable under my assumptions.
 
Posted by Orincoro (Member # 8854) on :
 
quote:
Originally posted by Tresopax:


Secondly, people often try to refute my position starting from assumptions that I don't agree with. In such cases, I have to say "I just don't believe that" because I don't. Please notice that I always at least try to give some explanation/reason why I don't believe it. But if I don't believe that shared definitions are unimportant to philosophers then I'm not going to go down a length path of discussion that is entirely built upon that assumption. If you want to convince me of your viewpoint, you need to start with assumptions I do believe in, or show me why I should accept the assumptions that I reject.

It would behoove you, for the sake of your relationship with others, or at least me, to keep in mind what is intended as an assumption for the sake of argument, and what is believed to be a common assumption. When you are reading someone else's argument, you can be perfectly justified in finding fault with it if it is not sound, however your serial offense IMO, is managing to simply not accept VERY basic assumptions so that no-one is able to argue with you. Notice that I don't hold you responsible for the existance of God in this thread, though I don't believe in God. If I were to follow what I believe is your strategy, I would merely dismiss your entire line of inquiry, ignoring all its other glaring faults, because I don't take the existance of God as a believable premise and therefore find the idea of ID absurd to the nth degree. Truth is, I don't believe in God, and so even if I could be convinced that your argument made ANY sense or that ID had anything to do with science, I STILL wouldn't pay it any mind because its based on a premise I don't believe. It happens to be a premise that it is not allowed to argue on this forum, iirc.

I could do this, and that would not a useful dialogue make, but that is what I see you doing time and again, and it is tiresome. Please, for my sake and for all involved, please consider this.
 
Posted by Orincoro (Member # 8854) on :
 
quote:
Originally posted by John Van Pelt:


So yes, there is a very general obligation on the scientific community to humbly allow that we don't know every dam-q thing, and that the universe undoubtedly has surprises in store for us. But that doesn't change one whit the characterization of what is practically considered the consensus body of knowledge, how it is taught, or the direction that scientists take in doing original research.

In other words, the "issue" you cite is indeed "small enough to neglect" (by a wide margin), and is thus no issue.

Edit: typo

But there is an issue here. Science will go on as before, because any scientist isn't going to waste time on a theory that is completely non-useful, unprovable, and not really a theory at all. What is at stake is education, and the fight by a religious group to mandate religious teachings in a public institution. This is a violation of the first ammendment, imo, and so it is a rights issue as well. It is really not a science issue, because science doesn't, and never will take this debate seriously as anything but an attack on education and rights.

The issue is that a religious group, fundamentalist Christians mainly, is attempting to legitimize its teachings and violate the rights of students and teachers by repackaging religious doctrines as scientific ideas. The fact that it isn't science is obvious, but we need to be sure that we don't spend public money, and allow public institutions to be a mouthpiece for religion of any kind. This stuff belongs in a church, funded by its members, privately.
 
Posted by John Van Pelt (Member # 5767) on :
 
Orincoro, Mathematician originally cited as an 'issue' the fact that scientists cannot really be 110% positive of their deductive conclusions, and that this left open a door (or ought to open a door) for seemingly non-scientific arguments to get SOME kind of hearing as alternatives to the leading accepted theories. My argument attempted to diminish that particular issue (to nothingness).

I certainly agree there is the different issue that you cite -- in fact, Mathematician's point of view seemed to be an example of it (albeit at its most benign).

I also agree pretty strongly with the rest of your opinions.

The only thing I would add -- while we are making these sweeping summaries [Smile] -- is that science, and science educators in particular, underestimate the degree of offense given religious familes and their children in science curricula. It is quite emotional. As long as that is the case, no amount of logic and education will clear up the dispute.

When I was being raised as a Christian Scientist, we were permitted 'accommodation' to skip health classes in school (Christian Scientists don't believe in conventional medical teachings or methods).

I've sometimes wondered if accommodation should be offered Fundamentalist Christian pupils, so that they can remove themselves from biology without penalty. But I don't think this works for lots of reasons, one being that if biology is being taught right, they'd have to be excused from pretty much the whole course. And that would disqualify them from many college programs, including pre-med. I don't think they would accept that.

If evolution is only being mentioned in one unit (and a recent survey revealed that in many biology classrooms across the country it isn't being mentioned at all) then I think we have deeper problems.
 
Posted by Tresopax (Member # 1063) on :
 
quote:
Your original point was that:
(1) during the ID debates, scientists claimed that the business of science and scientists was only that which could be proven via the scientific method (I am paraphrasing),
(2) a debate (undertaken by scientists) about the definition of the word 'planet' did not fall into that category,
(3) therefore the business of science and scientists must include a larger range of activities, including not just that which can be proven via the scientific method, but also the defining of terms, and perhaps other things,
(4) and therefore those scientists cited in (1) used an erroneous argument to disqualify discussions of ID from science.

Several of us replied with the news that can't have seemed anything but blindingly obvious to everyone but you, namely that the defining of terms (along with deciding what to eat for lunch and complaining about the weather) is something that everybody does all the time, is not a part of science per se, and whether a scientist engages in the defining of terms has no bearing whatever on how we may interpret what is the rightful domain of science and scientists.

Several posts later you say "I just don't think the discussion of which definitions are best should be considered doing science, even when the terms are scientific," which directly contradicts step (3) in my version of your argument above.

That does not contradict (3).

You are making an assumption here that if X is considered not "doing science" then X can't be the "business of scientists". I disagree with that assumption. In fact, that is my main point: The business of scientists includes MORE than just doing science.

quote:
Then you are wrong. Hypotheses and 'working theories' are assumptions -- and the expression I listed as an extrapolation is most definitely an extrapolation.
If that is how you take the term, then let me clarify: I don't think assumptions that aren't entailed by the results of testing, experimentation and observation should be considered science. Hypotheses and 'working theories' are a part of science though, because they are being tested, and are only considered true in a tentative way - they should not be considered 'fact' but rather as not yet proven false.

Of course, after something is tested millions of times there comes a point when it takes very little faith to jump from 'not yet proven false' to 'true as far as I'm concerned.' I do think that that leap is a leap beyond science, even if it is a very very small leap.

quote:
Science is useful in a number of different ways.
(1) Science as engineering has practical utility: we can build bridges, and they (usually) don't fall down.
(2) Life sciences including evolution, genetics, not to mention medicine, give us information about how the natural world works, and how we (as uniquely intelligent and uniquely destructive creatures) impact it and each other. This guides our behavior, if we let it, for good or ill.
(3) Scientific theories that are 'agreed' to be factual -- where there is strong consensus -- where you might say "we believe it's true" -- these have a special utility: they can be treated as axiomatic in the development of additional theories.

Actually, you are right. I was wrong on that point. Scientific theories can be useful even if you don't think they are definitely true.

quote:
When you are reading someone else's argument, you can be perfectly justified in finding fault with it if it is not sound, however your serial offense IMO, is managing to simply not accept VERY basic assumptions so that no-one is able to argue with you.
That is a serial offense of mine, but I disagree with you on its usefulness. I believe major differences in viewpoint stem from very basic assumptions, and so it is very basic assumptions that I like to discuss. But I don't just reject very basic assumptions without a reason given - if I were to reject the existence of God, I'd say why, and expect people to buy my point only insofar as they argeed with my reason.

I don't believe there are any assumptions that we aren't allowed to question on this forum.

Also, as a sidenote, I don't think anything I've said on this thread rests on the assumption that God exists. If it did, I think it would be valid for you to question that assumption - although in that case, we've discussed that particular assumption so many times that it might not be productive to do it again. Or it might be - that's up to you. [Smile]
 
Posted by John Van Pelt (Member # 5767) on :
 
quote:
Tres wrote:
that is my main point: The business of scientists includes MORE than just doing science.

So... since I believe everyone on this board (and probably everyone not on this board) would agree with that... of what special import is this point again? Maybe if we can start from scratch with this agreed basis, we can make some headway.
quote:
If that is how you take the term, then let me clarify: I don't think assumptions that aren't entailed by the results of testing, experimentation and observation should be considered science.
You are the one that started to use the term 'assumption,' not I. I merely clarified for you its meaning in a scientific context.

After two painful cycles of argument, you resolve the confusion by, in effect, saying, "I don't think assumptions that are derived or applied unscientifically should be considered scientific."

We're making great progress.
quote:
Hypotheses and 'working theories' are a part of science though, because they are being tested, and are only considered true in a tentative way - they should not be considered 'fact' but rather as not yet proven false.
This is wrong. There are billions of scientific discoveries, assertions, and conclusions that are positive, not tentative, and billions more -- perhaps an infinity -- that are far more than tentative.
quote:
Of course, after something is tested millions of times there comes a point when it takes very little faith to jump from 'not yet proven false' to 'true as far as I'm concerned.' I do think that that leap is a leap beyond science, even if it is a very very small leap.
This is a nice statement of some of the key issues embroiled in this debate, and it provides a really good segue.

My expression above, "far more than tentative," equates to the leap from "not yet proven false" to "true as far as science is concerned" (not "true as far as I'm concerned").

I wonder if this is really a fundamental aspect of how science is misunderstood by many including Tresopax. 'True as far as science is concerned' does NOT mean 'science says this is true', and it especially does not mean 'science says this is true (and it should be accepted as true by all people of all faiths and replace all related truths wherever they are).'

This is what I meant, I think, when I spoke above about science producing 'facts' but not 'truth.' A very few scientific areas approach the universality of truth -- most of mathematics, perhaps, but not much else.

'True as far as science is concerned' includes within it the tacit seeds of admission that:
'True as far as science is concerned' means:

 
Posted by Tresopax (Member # 1063) on :
 
quote:
So... since I believe everyone on this board (and probably everyone not on this board) would agree with that... of what special import is this point again?
I hope so - I like it when everyone agrees. [Smile] Just because everyone agrees on it, doesn't mean it isn't important to point out or remember.

Why is this point important? On the specific case of Intelligent Design, it refutes the argument that "ID is no business of scientists solely because it is not science" - an argument that I have heard made many times. But far far more importantly, it allows us to split scientific discussions into two areas - to distinguish between science itself and other claims that scientists make that are not directly supported by science itself. I think that if future generations are not taught this distinction clearly, the authority of science will inevitably be undermined as the line between science itself and religious/political/philosophical beliefs is blurred.

quote:
This is wrong. There are billions of scientific discoveries, assertions, and conclusions that are positive, not tentative, and billions more -- perhaps an infinity -- that are far more than tentative.
Name one scientific theory or model that is not tentative - that could not possibly turn out to be false. Even gravity might no longer work tomorrow.

quote:
I wonder if this is really a fundamental aspect of how science is misunderstood by many including Tresopax. 'True as far as science is concerned' does NOT mean 'science says this is true', and it especially does not mean 'science says this is true (and it should be accepted as true by all people of all faiths and replace all related truths wherever they are).'
Part of my trouble with this is I don't think all people realize this. I think many simply think "true as far as science is concerned" means it is actually true, certain, proven fact.
 
Posted by Destineer (Member # 821) on :
 
On an older thread about this, which people might want to check out (as it was very interesting), I posted the following:

quote:
There are other standards for telling science from pseudoscience besides complete falsifiability. In a 1981 court case regarding creation science, the following criteria were used:

(1) Explanation of existing results in terms of exceptionless natural laws.

(2) Ability to predict further results.

(3) Experimental testability to some extent.

(4) Tentativeness of central assumptions.

This is a more modern view of science than the "science only makes falsifiable claims" view that I often see Hatrackers assert.
 
Posted by Destineer (Member # 821) on :
 
And those who think Tres is closed-minded or obtuse should check out the old thread I linked to, because he totally came around to my view of science.
 
Posted by John Van Pelt (Member # 5767) on :
 
quote:
Tres wrote:
On the specific case of Intelligent Design, [the agreement that the business of scientists includes more than just doing science] refutes the argument that "ID is no business of scientists solely because it is not science"

Help me get this straight.

Even though ID is not science (and thus has no place in "doing science"), it can still have a place in the larger realm of "the business of science" (along with such things as deciding on the definition of the word planet).

Have I got it?

In which part of the definition would "deciding what should be taught in a science curriculum" fall: in "doing science" or "the business of science"? (Aside from other areas it might fall, such as "doing education", "doing public policy", etc.)
quote:
...it allows us to split scientific discussions into two areas - to distinguish between science itself and other claims that scientists make that are not directly supported by science itself.
I agree that this is an important distinction to maintain. For example, when scientists engage in political or social advocacy (e.g. climate change).

I would point out, however, that the activity of "distinguishing between science itself and other claims that scientists make that are not directly supported by science itself," is itself one of the things that scientists do when they are "doing science."

That's not to say that non-scientists -- citizens -- shouldn't also attempt to make that distinction. Science should be held accountable to intellectual and analytical rigor whatever its source.
quote:
Name one scientific theory or model that is not tentative - that could not possibly turn out to be false. Even gravity might no longer work tomorrow.
No fair picking on gravity! [Smile] Nobody knows what gravity is.

How about:
- Water molecules comprise two hydrogen atoms and one oxygen atom
- Between two sedimentary strata, the deeper one is the older (with certain exceptions easily elaborated in a fuller statement of the model)
- The moon orbits the earth, has a gravity about 1/6 of earth's, and is not made of green cheese
- E=MC2
- White light combines light of all colors in the visible spectrum
- a feather and a cannonball will fall the same distance in the same time in a vacuum
- some birth defects are due to genetic mutation, random errors in the sequence of a very complex molecule that is invisible to the naked eye.

I'm sure I could think of more if I put my mind to it.
quote:
I think many simply think "true as far as science is concerned" means it is actually true, certain, proven fact.
Many of your comments in these threads have given me the impression that you think Science itself (with a capital S) promotes that philosophy. It does not (IMO).
 
Posted by BaoQingTian (Member # 8775) on :
 
quote:
Originally posted by John Van Pelt:

I've sometimes wondered if accommodation should be offered Fundamentalist Christian pupils, so that they can remove themselves from biology without penalty. But I don't think this works for lots of reasons, one being that if biology is being taught right, they'd have to be excused from pretty much the whole course. And that would disqualify them from many college programs, including pre-med. I don't think they would accept that.

That's an interesting statement that kind of made me think. I don't know why fundamentalist Christians would want to be excused. Like it or not, evolutionary theory and its many extensions (genetics for example) are useful and actually work. I don't see why one could not learn the mechanisms of evolution while disagreeing with some of it's conclusions, such as man evolved from chimps if that's against your religion. Your explaination is better to you than the scientific one, so you might as well keep it. You don't need to completely agree with all the conclusions of such a theory for it to be useful to you.

It reminds me of the way I use i. Sure, there's no real number i, but that doesn't stop it from being useful in solving all sorts of engineering problems. If some of the potential implications of i were against my religious philosophy, I really don't have to buy into them for i to still be useful to me.
 
Posted by John Van Pelt (Member # 5767) on :
 
You're right, of course. I don't know why they would want to be excused, either. But it does answer the objection of really devout families for whom a teaching of geologic time and evolution is an offense against conscience -- without arbitrarily throwing in Creationist or ID palliatives that really don't belong in the (public school) curriculum.

Many such families already opt out -- the homeschool movement.
 
Posted by Destineer (Member # 821) on :
 
quote:
No fair picking on gravity! Nobody knows what gravity is.

Um, I do. It's curvature of the spacetime metric.

quote:
Sure, there's no real number i,
But there is a complex number i.

I'm always weirded out by the notion some people seem to have that the reals exist but the imaginaries don't.
 
Posted by BaoQingTian (Member # 8775) on :
 
quote:
But there is a complex number i.

I'm always weirded out by the notion some people seem to have that the reals exist but the imaginaries don't.

I'm not sure why my sentence is being misinterpreted by you. I flat out stated that i is not a real number. Which it is not. Although i exists in the COMPLEX number plane, you can't point it out to me on the REAL number line. I never said or implied that i did not exist.
 
Posted by Tarrsk (Member # 332) on :
 
quote:
Originally posted by Tresopax:
Why is this point important? On the specific case of Intelligent Design, it refutes the argument that "ID is no business of scientists solely because it is not science" - an argument that I have heard made many times.

Don't have time to respond in detail right now (oh, the perils of posting at work!), but, with all respect, it does not. At least, not in the way you're implying. It does perhaps refute the idea that "ID is no business of scientists," the first half of your statement, but since nobody said that to start with, I fail to see the relevance. If anything, scientists should be quite concerned with ID (IMO, anyway), and the political situation currently playing out, because it could have severe and dangerous ramifications for the future of biological research in this country. As for the second half ("...because it is not science"), it does not refute that at all. ID is not science because it claims explanatory power and yet cannot be falsified. Period.

I think we can agree that you've pointed out a hole in the blanket statement that "X is only scientific if it is falsifiable." But that was an overly simplistic description of science anyway, and certainly does not mean that "anything that is not falsifiable can be considered scientific." As I and others have pointed out many times in this thread, anything that purports to have explanatory power- that proposes a mechanism to account for what we observe- MUST be falsifiable. Both the theory of evolution and ID claim explanatory power. The word "planet" does not. Ergo, evolution and ID must be falsifiable to be considered scientific, whereas "planet" does not.

I know you delight in playing devil's advocate when it comes to basic assumptions, but the simple fact of the matter is that science IS based upon some simple assumptions: that the universe follows a set of rules, and that these rules don't change from one moment to the next. That's WHY scientific theories can have explanatory and predictive power. And science itself fulfills its own predictive criterion, as evidenced by the fact that bridges and skyscrapers don't spontaneously fly into space, the internal combustion engine works well enough to get millions of commuters to work each day, and vaccines prevent billions of deaths due to disease. If you don't accept the basic assumptions behind science, then I feel that we really don't have anything to discuss.
 
Posted by Destineer (Member # 821) on :
 
quote:
I'm not sure why my sentence is being misinterpreted by you. I flat out stated that i is not a real number. Which it is not. Although i exists in the COMPLEX number plane, you can't point it out to me on the REAL number line. I never said or implied that i did not exist.
Well, you were making an analogy with someone who might deny that evolution occurred, and yet still use it in science. So I assumed you meant that you might (or actually do) deny that i is a number, and yet still use it in math.

If that's not what you're saying, the analogy seems less than apt.
 
Posted by Orincoro (Member # 8854) on :
 
Destineer, you are trying to pick apart a person's thoughts on a scientific theory based on how the same person uses mathematical concepts. You aren't going to a) achieve a comprehensible correlation between unreal numbers and scientific theories, or b) use that correlation to convince us of anything.

You are attacking a mathematical principle which yeilds positive results (no pun intended), when really you will only strengthen Mathematician's point by showing us how theories can be so useful. This is very sketchy argumentation and you know it, I wouldn't go any further with it, but that is the hatrack way I suppose.
 
Posted by Mathematician (Member # 9586) on :
 
quote:
Originally posted by Destineer:
quote:
No fair picking on gravity! Nobody knows what gravity is.

Um, I do. It's curvature of the spacetime metric.

I'd gather from this you don't know what gravity is. The spacetime metric is a function acting on two vectors with certain properties. An equation cannot be curved, at least in the general relativistic sense. What can be curved is the manifold on which all these equations live.

As a quick example, consider this function: Given any place on earth, the function f tells you the latitude of that place. Is the function f curved? What does that even mean? Is 2+2 = 4 curved?

Is the thing the function acts on curved? Sure, the set of all points on the earth forms a curved manifold.

Same idea with gravity.


Beyond even this though, Einstein came up with a great mathematical description of gravity. But that certainly doesn't mean we must all actually exist on a 4-dimensional manifold. In otherwords, Einstein did a great job of giving us equations to model gravity, but he gave us nothing to say "this is what gravity REALLY is"


quote:
Originally posted by Desineer:
quote:
Sure, there's no real number i,
But there is a complex number i.

I'm always weirded out by the notion some people seem to have that the reals exist but the imaginaries don't.

Me too.
 
Posted by King of Men (Member # 6684) on :
 
Let me just chime in and say that Mathematician is quite right about gravity; redefining it as 'curvature of the spacetime metric' is just playing with words. Even if you actually understand the equations, you're still only playing with numbers. You have gained something in your ability to describe what gravity does, which is why playing with numbers is a useful activity, but you haven't said anything about what it is. Nor does a scientist care; the proper aim is to get your ICBM to hit the right place, never mind the philosophical claptrap. There's a reason 'metaphysics' is an insult.
 
Posted by Orincoro (Member # 8854) on :
 
But KoM, aren't scientists interested in the extistance of gravitons as a possible element in the eventuall testing of M theory? I am so an amatuer/hobby reader, but I think scientists would at least care to know what gravity "is" if it is indeed something like a particle or a wave, right?

edit: although, there we are still talking about how it behaves and not what it is.. but aren't you at least curious about what it is? The lady or the graviton?
 
Posted by Mathematician (Member # 9586) on :
 
quote:
Originally posted by Orincoro:
But KoM, aren't scientists interested in the extistance of gravitons as a possible element in the eventuall testing of M theory? I am so an amatuer/hobby reader, but I think scientists would at least care to know what gravity "is" if it is indeed something like a particle or a wave, right?

edit: although, there we are still talking about how it behaves and not what it is.. but aren't you at least curious about what it is? The lady or the graviton?

Sure they would like to know what it is, but the point is that when you start getting to the heavy math subjects, it's really just that - heavy math.

Quantum Mechanics and General Relativity are very mathematical theories, string theory more so. Thus, an extremely rational person wouldn't say "we live on a curved 4-manifold", but "General Relativity wonderfully predicts the effects of gravity using the mathematics of curved manifolds as a model of the universe"

Likewise, one doesn't say "An electron is both a particle and a wave", but ,"Quantum mechanics best explains the behavior of an electron by assuming it can take on both wave-like properties and particle-like properties"

Or to use your photon example, photon is just short hand for a "packet of light", which is just a ripple in an electromagnetic field. Do electromagnetic fields REALLY exist? Or are they just convenient mathematical structures for accurately predicting behavior?

Finally, an exremely rational person would say, "String theory says the best model of the universe comes from assuming that everything is made up of bands of energy vibrating in different dimensions", as opposed to, "String theory says every particle is made out of tiny strings of energy"


I guess to summarize, especially with mathematics intensive fields, until you can observe something, the best you can say is "this math models what I'm seeing better than any other math I've ever seen", or in some good cases, "this is the best possible mathematical model matching all of the assumptions". One cannot say "this mathematical model works perfectly and therefore this is how the universe really is."
 
Posted by Destineer (Member # 821) on :
 
quote:
I'd gather from this you don't know what gravity is. The spacetime metric is a function acting on two vectors with certain properties. An equation cannot be curved, at least in the general relativistic sense. What can be curved is the manifold on which all these equations live.
Sorry about the slightly imprecise language. That said, you can have a manifold without a metric, and the metric is what determines the curvature of the manifold. And the Einstein field equations directly govern the metric tensor, so really it's facts about the metric (and not about the points) that determines the effects of gravitation.

Besides which, facts about the points of the manifold aren't in general diffeomorphism-invariant, so on the most common understanding of GR, they count as unobservable gauge.

quote:
Let me just chime in and say that Mathematician is quite right about gravity; redefining it as 'curvature of the spacetime metric' is just playing with words. Even if you actually understand the equations, you're still only playing with numbers. You have gained something in your ability to describe what gravity does, which is why playing with numbers is a useful activity, but you haven't said anything about what it is. Nor does a scientist care; the proper aim is to get your ICBM to hit the right place, never mind the philosophical claptrap. There's a reason 'metaphysics' is an insult.
I think this view is profoundly mistaken. For one thing, there's no strong distinction to be drawn between theoretical input and experimental output. This distinction is basically a function of what the experimenter takes for granted, rather than any fundamental difference between measurable and non-measurable things. So for instance, a physicist using electron diffraction to measure a crystal will treat the position of the electrons as 'known' input, whereas another physicist studying the behavior of electrons will treat the same data as the unkown quantity to be measured.

Further, and perhaps more importantly, what we can measure depends on which theory of our own sensory organs and instruments is true. There's no way to measure anything without relying on theoretical assumptions about what you can measure.

This means the only sensible alternative is to accept, tentatively, the approximate truth of everything your best science tells you. Not just the 'experimental results' -- because the theory itself can't tell you which facts are experimental.

Bringing this back to relativity: relativity has taught us that spacetime exists. It's measurable, just like protons and dogs are measurable. I perceive it every time I feel the pull of gravity. There's no other explanation for effects like gravitational lensing, besides the apparent fact that objects exist in a curved, four-dimensional continuum. So I must assume that they really do, at least until better science (eg strings?) indicates otherwise.
 
Posted by Destineer (Member # 821) on :
 
quote:
I guess to summarize, especially with mathematics intensive fields, until you can observe something, the best you can say is "this math models what I'm seeing better than any other math I've ever seen", or in some good cases, "this is the best possible mathematical model matching all of the assumptions". One cannot say "this mathematical model works perfectly and therefore this is how the universe really is."
I agree, obviously, that we should be tentative. But I think mathematical models, just like any other inferences we make from evidence, should be treated as our best guess about how the world is.

quote:
Or to use your photon example, photon is just short hand for a "packet of light", which is just a ripple in an electromagnetic field. Do electromagnetic fields REALLY exist? Or are they just convenient mathematical structures for accurately predicting behavior?
This is another good example of what I'm talking about. If the EM field is a real, physical field and not just a mathematical construct, then its presence at every point of space explains why charges move the way they do. If it's not a real field, then there's nothing out there in the world telling the charges what to do. So why do opposite ones attract rather than repel?

If you believe the EM field exists, you have an answer. Otherwise you don't. That's why I believe in the EM field, at least at the classical level.

[ September 06, 2006, 10:09 AM: Message edited by: Destineer ]
 
Posted by Destineer (Member # 821) on :
 
By the way, Mathematician, glad to have another geometry junkie on the board. [Smile]
 
Posted by BaoQingTian (Member # 8775) on :
 
quote:
Originally posted by Destineer:
Well, you were making an analogy with someone who might deny that evolution occurred, and yet still use it in science. So I assumed you meant that you might (or actually do) deny that i is a number, and yet still use it in math.

If that's not what you're saying, the analogy seems less than apt.

For future reference, don't try to read to much into my analogies. They are usually hastily constructed to make a superficial point, start taking them too deeply or broadly and they'll stop making any sort of sense. My main error was talking about the way I use i and then introducing a hypothetical person that has a problem with the existence of the number-for clarity I probably should have just kept the whole analogy hypothetical.

Or perhaps rather than hypothetical, just put Descartes into the analogy, at it should work a little better for you.
 
Posted by orlox (Member # 2392) on :
 
Watching scientists grapple with philosophy is really quite entertaining.

From E.P. Thompson:

In the old days (one supposes) when the philosopher, labouring by lamp-light in his study, came to this point in his arguement, he set down his pen, and looked around for an object in the real world to interrogate. Very commonly that object was the nearest one to hand: his writing table. "Table," he said, "how do I know that you exist, and, if you do, how do I know that my concept, table, represents your real existence?" The table would look back without blinking, and interrogate the philosopher in its turn. It was an exacting exchange, and according to which one was the victor in the confrontation, the philosopher would inscribe himself as idealist or as materialist.
 
Posted by rivka (Member # 4859) on :
 
quote:
Originally posted by Mathematician:
I have read books/articles by Gerald Schroeder and by some other guy (I don't remember his name) which address this.

Oy, gevalt. Please don't cite Schroeder -- to quote my father (a mathematical physicist and an Orthodox Jew), his book are "bad physics and bad Torah."

As for "the other guy," I suspect you are talking about Larry Keleman, whose books are great (and you should hear him in person!) I also recommend Nosson Slifkin's books, although I disagree with him on a fair number of details.
 
Posted by King of Men (Member # 6684) on :
 
quote:
So for instance, a physicist using electron diffraction to measure a crystal will treat the position of the electrons as 'known' input, whereas another physicist studying the behavior of electrons will treat the same data as the unkown quantity to be measured.
Totally irrelevant. I refer you back to the example I mentioned, of making an ICBM hit the right spot. Technology forms an independent test of science; if your lightbulb works, your science is right enough. It can get better, so that for example you are able to get a radio to work without losing the lightbulb. But the distinction between 'known input' and 'experimental result' is irrelevant; the question is whether you can make a better killing machine.
 
Posted by Destineer (Member # 821) on :
 
quote:
Technology forms an independent test of science; if your lightbulb works, your science is right enough.
What about science with no known technological application, like high-energy particle physics or cosmology? How does your standard of rightness (or 'right enoughness') apply to these situations?

Also, I would maintain that even in the ICBM case, your criterion of success is theory-dependent. How is it that you assert that the ICBM hit the right target? Well, you have a theory of motion that tells you what it is for a missile to collide with a target, and a theory of optics that tells you how the collision is visible to the naked eye.

These theories might be ill-defined 'folk theories' that you formed from everyday life rather than experimental science. But they're theories nonetheless.
 
Posted by Mathematician (Member # 9586) on :
 
quote:
Originally posted by rivka:
quote:
Originally posted by Mathematician:
I have read books/articles by Gerald Schroeder and by some other guy (I don't remember his name) which address this.

Oy, gevalt. Please don't cite Schroeder -- to quote my father (a mathematical physicist and an Orthodox Jew), his book are "bad physics and bad Torah."

As for "the other guy," I suspect you are talking about Larry Keleman, whose books are great (and you should hear him in person!) I also recommend Nosson Slifkin's books, although I disagree with him on a fair number of details.

Thanks for the heads up about Schroeder. I knew he was bad at physics, but I had no idea he screwed up the "Torah" side. What do you know about this Nachmanides guy Schroeder seems to quote a lot?

All this said, I still find his writings interesting (though not science or Torah) ;-)

As far as the other 2 guys you mentioned, I don't THINK that's them, but I really don't remember.

[ September 09, 2006, 11:29 PM: Message edited by: Mathematician ]
 
Posted by rivka (Member # 4859) on :
 
Nachmanides, more commonly known as the Ramban (not to be confused with the Rambam, an approximate contemporary with who he disagreed on many points but respected greatly), was a great Torah scholar. His commentary on Chumash (Five Books of Moses) is considered one the five (or so) most basic commentaries on the text, and included in all Mikraos Gedolos (essentially the gold standard for printed volumes of Tanach) Chumashim.
 
Posted by Mathematician (Member # 9586) on :
 
rivka,

Thanks a lot.


just so we're on the same page, "5 books of Moses" is Genesis, Exodus, Leviticus, Numbers, and Deuteronomy? I guess what I'm really asking is about Deuteronomy. As Moses dies in it, he probably didn't write all of it ;-). Is there another book which is authored by Moses (which perhaps, didn't make it into the Christian Old Testament Canon?)

Also, what's "Tanach"?

Finally, when saying Ramban's commentary "is one of the 5 or so most basic commentaries on the text", what exactly do you mean by basic? Basic meaning one of the 5 or so standard starting places when looking up Torah commentary?

Thanks again!
 
Posted by King of Men (Member # 6684) on :
 
quote:
Originally posted by Destineer:
quote:
Technology forms an independent test of science; if your lightbulb works, your science is right enough.
What about science with no known technological application, like high-energy particle physics or cosmology? How does your standard of rightness (or 'right enoughness') apply to these situations?
Particle physics has applications to fusion power, cancer cures, and breaking up radioactive waste. Further, it makes predictions about what our detectors are going to show; and those detectors can be checked elsewhere, against the usual technological 'right enough' standard. It's all a web, but ultimately it refers back to lightbulbs.


quote:
Also, I would maintain that even in the ICBM case, your criterion of success is theory-dependent. How is it that you assert that the ICBM hit the right target? Well, you have a theory of motion that tells you what it is for a missile to collide with a target, and a theory of optics that tells you how the collision is visible to the naked eye.
That's stretching the word 'theory' into utter meaninglessness, and I won't go there.

[ September 10, 2006, 03:31 PM: Message edited by: King of Men ]
 
Posted by rivka (Member # 4859) on :
 
quote:
just so we're on the same page, "5 books of Moses" is Genesis, Exodus, Leviticus, Numbers, and Deuteronomy?
Yes, although that's not what I call them. [Wink]
quote:
I guess what I'm really asking is about Deuteronomy. As Moses dies in it, he probably didn't write all of it ;-).
That is debated, actually. Traditional possibilities:

Regardless, all but the last 8 verses have no such difficulty.
quote:
Also, what's "Tanach"?
Acronym for Torah (5 books), Nevi'im (Prophets), and Kesuvim (Writings). Essentially what a Christian would call the "Old Testament."
quote:
Finally, when saying Ramban's commentary "is one of the 5 or so most basic commentaries on the text", what exactly do you mean by basic? Basic meaning one of the 5 or so standard starting places when looking up Torah commentary?
Yup.
 
Posted by rivka (Member # 4859) on :
 
quote:
Originally posted by Mathematician:
As far as the other 2 guys you mentioned, I don't THINK that's them, but I really don't remember.

Was it by any chance Aviezer? I haven't read that one, but as I recall my father considers it "not terrible." (Other people I know are more effusive. [Wink] )
 
Posted by John Van Pelt (Member # 5767) on :
 
quote:
That's stretching the word 'theory' into utter meaninglessness, and I won't go there.
I agree, but I'd already been thinking that KoM's answer to 'the nature of science' ("Nor does a scientist care; the proper aim is to get your ICBM to hit the right place, never mind the philosophical claptrap") threatened to stretch (or constrain) the word 'science' to utter meaninglessness.

I wasn't orginally going to respond (with more of my characteristic claptrap, sorry), but I'm curious about KoM's latest.

Is technology the aim of science? Is it merely a convenient, and independent, 'test' of science? Is it part of science?

And more to the point, what is technology? "If the field is irrigated, the theory about Archimedes screws is right enough." Obviously an infinite number of examples, from rubbing two sticks together to annihilating the globe, can be concocted to mirror this statement. But just as obviously, it's important to HAVE the right theory (or a right-enough theory) -- independent of experimental results -- in order to derive spin-off conjectures or to connect the underlying prcinciple of one physical phenomenon with that of another.

You may say, well that will just result in another light-bulb test, which will fail or succeed according to the correct-enoughness of the spin-off conjecture. True -- but the link between the two experiments, as exemplified in the scientist's advanced understanding of underlying principles/math/whatever cannot be thus consigned to irrelevance.

But maybe I am misunderstanding your point.

In addition, there is a vast amount of science (maybe even most, certainly most life science) for which analogues of "the proper aim is to get your ICBM to hit the right place" are hard if not impossible to identify.

Maybe the notion that science is about knowledge, as well as results, is a little metaphysical. Then again, I don't consider metaphysics a dirty word.
 
Posted by Destineer (Member # 821) on :
 
quote:
Particle physics has applications to fusion power, cancer cures, and breaking up radioactive waste.
Not all particle physics has such applications. I doubt Higgs bosons or top quarks will be used in radiation therapy or fusion any time soon.

In fact, due to confinement, quark color is an unobservable quantity with no possible application. That's provable from the form of QCD, although some of the proofs are a bit fuzzy. And you didn't address cosmology in your response. Bottom line, I think I've shown that you're wrong that all science has technological applications.

quote:
Further, it makes predictions about what our detectors are going to show; and those detectors can be checked elsewhere, against the usual technological 'right enough' standard.
Really? How do you check the performance of a neutrino detector, for instance, except by detecting neutrinos with it?

That's right, neutrinos... another particle with zero technological application.

quote:
That's stretching the word 'theory' into utter meaninglessness, and I won't go there.

Newtonian mechanics isn't a theory? Because that's what explains collisions. Geometric optics isn't a theory? Because that's what explains how the eye works.
 


Copyright © 2008 Hatrack River Enterprises Inc. All rights reserved.
Reproduction in whole or in part without permission is prohibited.


Powered by Infopop Corporation
UBB.classic™ 6.7.2